You are on page 1of 70

ForumIAS

Prelims Marathon
March
2018

HISTORY
ECONOMICS
POLITY
SCIENCE AND TECHNOLOGY
GEOGRAPHY AND ENVIRONMENT
PRELIMS MARATHON COMPILATION FOR THE MONTH OF MARCH, 2018

History
Q.1) With reference to the Benares session (1905) of Indian National Congress, consider the following
statements:
1. This session was presided by G.K Gokhale.
2. At this session, self-government or Swaraj was declared as the goal of the Indian National Congress.
3. At this session, the leaders of Congress unanimously decided to extend the Swadeshi movement
outside Bengal.
Which of the above statements is/are correct?
a) 1 only
b) 1 and 2 only
c) 2 and 3 only
d) 1, 2 and 3

Q.2) Consider the following statements about Dharsana Satyagraha:


1. It aimed to oppose the salt tax.
2. It was lead by Sarojini Naidu.
3. It was launched during the civil disobedience movement.
Which of the statement given above is/are correct?
a) 2 only
b) 1 and 2 only
c) 2 and 3 only
d) 1, 2 and 3

Q.3) Which of the following were the reasons that led to withdrawal of Swadeshi movement?
1. Split in Indian National Congress
2. Repression by the government
3. Arrest and deportation of major leaders.
4. Annulment of partition of Bengal
Select the correct answer using code given below.
a) 2 and 3 only
b) 1, 2 and 3 only
c) 1 and 4 only
d) 1, 2, 3 and 4

Q.4) Which of the following party/parties was/were established by Dr. Bhimrao Ramji Ambedkar?
1. Bahishkrit Hitakarini Sabha
2. Independent Labour Party (ILP)
3. Peasants and Workers Party of India
Select the correct answer using the code given below.
a) 1, 2 and 3
b) 1 and 2 only
c) 1 and 3 only
Created with by ForumIAS.com – The Knowledge Network for Civil Services.
Visit http//forumias.com New! http://forumiasacademy – Prelims & Mains Test Series

ForumIAS
PRELIMS MARATHON COMPILATION FOR THE MONTH OF MARCH, 2018

d) 3 only

Q.5) Which among the following founded the Servants of India Society?
a) Jyotiba Phule
b) Sri Narayan Guru
c) B.R. Ambedkar
d) Gopal Krishna Gokhale

Q.6) Which of the following were the results of the revolt of 1857?
1. The assumption of the Government of India by the sovereign of Great Britain.
2. The beginning of the era of annexations and expansion.
3. The recognition of the paramountcy of the British Crown by the Indian states.
4. The reformation of British of military policy.
Select the correct answer using the code given below.
a) 1, 2 and 3
b) 2 and 3 only
c) 1, 3 and 4 only
d) All of the above

Q.7) Consider the following pairs:


Leader during the revolt of 1857 Centre
1. Khan Bahadur: Bareilly
2. Maulvi ahmadullah: Faizabad
3. Rani Laxmibai: Jhansi
Which of the pairs given above is/ are incorrectly matched?
a) 1 and 2 only
b) 1 only
c) 2 and 3
d) None of the above

Q.8) Consider the following statements:


1. The practice of child marriage and polygamy was absent in the Vedic society.
2. The induction of new officials like purohita, senani, gramani etc. was started during the later Vedic
period.
Which of the above statements is/are correct?
a) 1 only
b) 2 only
c) Both 1 and 2
d) Neither 1 nor 2

Created with by ForumIAS.com – The Knowledge Network for Civil Services.


Visit http//forumias.com New! http://forumiasacademy – Prelims & Mains Test Series

ForumIAS
PRELIMS MARATHON COMPILATION FOR THE MONTH OF MARCH, 2018

Q.9) Consider the following statements:


1. The Vijaynagara School of Architecture was completely indigenous in nature.
2. The sculptures on the pillars were carved with animals and the horse was the most common among
them.
Which of the following statements is/are correct?
a) 1 only
b) 2 only
c) Both 1 and 2
d) Neither 1 nor 2

Q.10) With reference to the cultural history of India, the terms 'Lavanyayoganam' and 'Pramanam', were
a) Style of temple construction
b) Administrative functionaries mentioned in the Arthashastra
c) Principles or limbs of paintings
d) Dresses popular among the women of royal and noble families

Q.11) Which among the following discovered the paintings at bhimbetaka?


a) John Hubert Marshall
b) Vishnu Wakankar
c) Madho Sarup
d) Rai Bahadur Daya Ram Sahni

Q.12) Consider the following historical places:


1. Bagh caves
2. Ravan Chhaya Rock shelter
3. Sanchi Stupa
Which of the above places is / are also known for mural paintings?
a) 1 only
b) 2 and 3 only
c) 1 and 2 only
d) None of the above

Q.13) With reference to Carnatic music, consider the following statements:


1. It does not have sub-styles.
2. Tabla is one of the instruments associated with it.
3. It is indigenous in nature.
Which of the above statements is/are correct.
a) 1, 2 and 3
b) 2 and 3 only
c) 1 and 3 only
d) None of the above is correct

Created with by ForumIAS.com – The Knowledge Network for Civil Services.


Visit http//forumias.com New! http://forumiasacademy – Prelims & Mains Test Series

ForumIAS
PRELIMS MARATHON COMPILATION FOR THE MONTH OF MARCH, 2018

Q.14) Gombayetta is a famous traditional puppets belongs to


a) Madhya Pradesh
b) Karnataka
c) Gujarat
d) Andhra Pradesh

Q.15) Which of the following initiative(s) was/were undertaken by M.K. Gandhi during his stay in
Africa?
1. Establishment of Natal Indian Congress.
1. Campaign against restriction on Indian migration.
2. Campaign against poll tax and invalidation of Indian marriages.
Select the correct answer using the code given below.
a) 1 and 2 only
b) 1 only
c) 2 and 3 only
d) 1, 2 and 3

Q.16) The caves and rock-cut temples at Ellora are associated to


a) Hindu and Jain
b) Buddhist
c) Buddhist and Jain
d) Hindu, Buddhist and Jain

Q.17) "At this session of Indian National Congress, a demand was made for permanent fixation of land
revenue. This session was presided by Romesh Chandra Dutt."
Which among the following session of Indian National Congress has been described in the above
passage?
a) First Lucknow session
b) First Madras session
c) Second Allahabad session
d) Second Madras session

Q.18) With reference to the aims of early nationalists in the 19th century, consider the following
statements:
1. They aimed in creation of national unity.
2. They wanted to achieve the freedom through direct struggle.
Which of the above statement is/are correct?
a) 1 only
b) 2 only
c) Both 1 and 2
d) Neither 1 nor 2

Created with by ForumIAS.com – The Knowledge Network for Civil Services.


Visit http//forumias.com New! http://forumiasacademy – Prelims & Mains Test Series

ForumIAS
PRELIMS MARATHON COMPILATION FOR THE MONTH OF MARCH, 2018

Q.19) Which of the following statements is/are correct about the 'Rajagopalachari formula'?
1. It wanted to solve the constitutional crisis occurred due to the failure of the August offer.
2. Its proposals were shadowed by the two nation theory of Muslim League.
Select the correct answer using the code given below.
a) 1 only
b) 2 only
c) Both 1 and 2
d) Neither 1 nor 2

Q.20) Which of the following was/were proposal/proposals brought by the Wavell Plan?
1. All members of the executive council were to be Indians except the Governor –General.
2. Caste Hindus and Muslims were to have equal representation.
3. Governor-General was to exercise his veto power on the advice of ministers.
Select the correct answer using the code given below.
a) 1 only
b) 2 and 3 only
c) 1 and 2 only
d) 1, 2 and 3

Q.21) At which session of the Indian National Congress the word ‘swaraj was mentioned for the first
time?
a) Fifth Calcutta session
b) Banaras session
b) Third Calcutta Session
c) Second Madras

Q.22) Which of the following factor(s) was/were behind in popular support for Home rule league?
1. The Failure of the Congress to launch mass movement under moderate leadership.
2. The world war destroyed the myth of the racial superiority of colonial powers.
3. The imposition of heavy tax and high prices of commodities during the world war.
Select the correct answer using the code given below.
d) 1, 2 and 3
a) 1 only
b) 2 and 3 only
c) 1 and 3 only

Q.23) With reference to various popular resistance movements against the British government, consider
the following statements:
1. All resistance movements were local in character and effects.
2. The resistance movements of zamindars and rulers were violent in nature while the tribal resistances
were non-violent.
3. Displaced zamindars and rulers wanted to regain their land and estates while the tribal groups rebelled
Created with by ForumIAS.com – The Knowledge Network for Civil Services.
Visit http//forumias.com New! http://forumiasacademy – Prelims & Mains Test Series

ForumIAS
PRELIMS MARATHON COMPILATION FOR THE MONTH OF MARCH, 2018

mostly against the traders and money lenders’ interference in their lives.
Which of the statements given above is/are correct?
a) 1 and 3 only
b) 1 and 2 only
c) 2 and 3 only
d) 1, 2 and 3

Q.24) Which of the following statements is/are incorrect about the Government of India Act of 1935?
1. It created diarchy at the centre as well as in the provinces.
2. It provided an all India federation with two type of government i.e. Union and State.
3. It gave freedom to native states either to join or quit the Federation.
Select the correct answer using the code given below.
a) 2 only
b) 1 and 2 only
c) 1 only
d) 1, 2 and 3

Q.25) Consider the following events related to the later phase of Indian freedom struggle:
1. The Quit India Movement
2. The August offer
3. The Royal Indian Naval Ratings Revolt
4. The INA Trial
Which among the following is the correct sequence as per their occurrence?
a) 2, 4, 1, 3
b) 2, 1, 4, 3
c) 1, 2, 3, 4
d) 1, 4, 2, 3

Q.26) Which of the following statements is/are correct about Indian press during the British period?
1. Pre-censorship was one of the dominant features of British press policy throughout their tenure in
India.
2. All publishers are required to have a specific premise of publication under the Press Act of 1835.
3. Both the Vernacular Press Act of 1878 and Indian Press Act of 1910 imposed a security deposits on
Indian press.
Select the correct answer using the code given below.
a) 1 only
b) 2 and 3 only
c) 1 and 2 only
d) 1, 2 and 3

Q.27) Consider the following statements:


1. Both Vedic and Sangam texts are religious in nature.
2. Unlike Vedic texts, Sangam texts do not provide any information on the political and economic life of
people
Which of the statements given above is/are incorrect?
a) 1 only
Created with by ForumIAS.com – The Knowledge Network for Civil Services.
Visit http//forumias.com New! http://forumiasacademy – Prelims & Mains Test Series

ForumIAS
PRELIMS MARATHON COMPILATION FOR THE MONTH OF MARCH, 2018

b) 2 only
c) Both 1 and 2
d) Neither 1 nor 2

Q.28) Consider the following languages:


1. Odia
2. Tamil
3. Bengali
4. Telugu
Which of the above have been declared as ‘Classical Languages’ by the Government of India?
a) 1, 2 and 3 only
b) 1 and 2 only
c) 3 and 4 only
d) 1, 2 and 4 only

Q.29) Consider the following statements:


1. The lady representative of India at the Second Round Table Conference was Vijayalakshmi Pandit.
2. The Indian National Congress and the Muslim League did not send any representative to the First
Round Table conference.
3. The recommendations of third Round Table Conference were published in a White Paper in 1933 and
later discussed in the British Parliament.
Which of the statements given above is/are correct?
a) 1 and 2 only
b) 3 only
c) 1 and 3 only
d) 1, 2 and 3

Answer Key

Qs. Ans Qs. Ans Qs. Ans


1 A 11 B 21 A
2 D 12 C 22 A
3 B 13 C 23 A
4 B 14 B 24 C
5 D 15 D 25 B
6 C 16 D 26 B
7 D 17 A 27 C
8 D 18 A 28 D
9 B 19 B 29 B
10 C 20 B

Created with by ForumIAS.com – The Knowledge Network for Civil Services.


Visit http//forumias.com New! http://forumiasacademy – Prelims & Mains Test Series

ForumIAS
PRELIMS MARATHON COMPILATION FOR THE MONTH OF MARCH, 2018

Explanation
1. Statement 1 is correct. The Indian National Congress Benares session was presided over by G.K.
Gokhale.
Statement 2 is incorrect. The goal of the Indian National Congress as 'self-government or Swaraj like that
of the United Kingdom or the Colonies' was declared at the Calcutta session (1906). It was presided by
Dadabhai Naoroji.
Statement 3 incorrect. At the Benaras session of the Indian National Congress presided over by Gokhale,
the Moderate-Extremist differences came to the fore. The Extremists wanted to extend the Boycott and
Swadeshi Movement to regions outside Bengal and also to include all forms of associations (such as
government service, law courts, legislative councils, etc.) within the boycott programme and thus start a
nationwide mass movement.
The Extremists wanted a strong resolution supporting their programme at the Benaras session. The
Moderates, on the other hand, were not in favour of extending the movement beyond Bengal and were
totally opposed to boycott of councils and similar associations. They advocated strictly constitutional
methods to protest against the partition of Bengal. As a compromise, a relatively mild resolution
condemning the partition of Bengal and the reactionary policies of Curzon and supporting the Swadeshi
and boycott programme in Bengal was passed. This succeeded in averting a split for the moment.

2. All the statements are correct. Dharasana Satyagraha was a protest against the British salt tax in
colonial India in May, 1930. Sarojini Naidu, Imam Saheb and Gandhiji's son Manilaal participated in
Dharasana salt Satyagraha. It was launched during civil disobedience movement.

3. The Swadeshi Movement fizzled out by 1908 due to the following reasons:
1. Severe government repression.
2. Lack of effective organisation and a disciplined focus.
3. With arrest, deportation of all leaders, the movement left leaderless.
4. Split in nationalist ranks.
5. Narrow social base.
Annulment of Partition of Bengal was done in 1911 in Delhi Durbar after waning away of Swadeshi
movement. Thus, statement 4 is incorrect.

4. Dr. Ambedkar founded Bahishkrit Hitakarini Sabha and Independent Labour Party (ILP).
The Peasants and Workers Party of India was founded in 1947. The party was founded in Maharashtra by
Tulsidas Jadhav, Keshavrao Jedhe and others.

5. Gopal Krishna Gokhale was the founder of the Servants of India Society. He wanted to build a
dedicated group of people for social service and reforms. In the field of famine relief, union organisation,
cooperatives and uplift of tribals and depressed, the Society did commendable work.

6. All the statements are correct except statement 2. The revolt of 1857 marks a turning point in the
history of India. It led to changes in the system of administration and the policy of the Government.
As a result of the revolt, the era of annexations and expansion ended and the British promised to respect
the dignity and rights of the native princes.
Created with by ForumIAS.com – The Knowledge Network for Civil Services.
Visit http//forumias.com New! http://forumiasacademy – Prelims & Mains Test Series

ForumIAS
PRELIMS MARATHON COMPILATION FOR THE MONTH OF MARCH, 2018

7. All the pairs are correctly matched.

8. Statement 1 is incorrect. In Vedic society, monogamy was generally practiced while polygamy was
prevalent among the royal and noble families. There was no child marriage and the practice of sati was
absent.
Statement 2 is incorrect. In the later Vedic period, a large number of new officials were involved in the
administration in addition to the existing purohita, senani and gramani. They include the treasury officer,
tax collector and royal messenger. At the lower levels, the administration was carried on by the village
assemblies.

9. Statement 1 is incorrect. The rulers of Vijaynagara Empire combined the features of chola, Hoysala,
Pandyas, Chalukyas architecural styles. Under them, the architectural style started to be influenced by
the Indo-Islamic style of Bijapur, which in turn was reflected in the temples built during the period of
1335-1565 AD.
Statement 2 is correct. The sculptures on the pillars were carved with distinctive features. The horse was
the most common animal found in these pillars.

10. Vatsyayana in his book mentioned about 6 main principles/limbs or shadanga of paintings they are:
Rupabheda (Variety of form), Sadrisyan (portrayal of likeliness of the subject), Bhava (creation of luster
and gleam with the colours), Varnikabhanga (mixing of colours to resemble the effects of modelling)
pramanam (proportion of the object or subject) and Lavanyayoganam (immersion of emotions).

11. The caves of Bhimbetka were discovered by an archaeologist from Vikram University, Ujjain, Dr.
Vishnu Wakankar. He strayed too far from the beaten path and found himself amidst this prehistoric
treasure trove. In due course of time, the archaeological excavations yielded remains, serially from the
lower palaeolithic Age to the early medieval Ages.

12. The Stupa at Sanchi does not have mural paintings. The works on the walls or a solid structure are
referred to as Murals. The evidence of such paintings can be found at several locations in India. Some of
locations are Ajanta, Armamalai cave, Ravan Chhaya rock Shelter, Bagh caves, sittanavasal caves and
Kailasanatha temple in Ellora.
The Stupa at Sanchi is one of the most important Buddhist monuments reflecting gem of Buddhist art and
architecture. This consists of a central chamber where the relics of Lord Buddha are placed. Four
ornamental gateways facing four directions and a balustrade surrounding the Stupa were later added in
the first century BCE. A typical example of a Stupa and an excellent illustration of the development of
Buddhist art and sculpture starting from the third century BC through the twelfth century AD. Enlisted
as a UNESCO World Heritage Site since 1989, it is counted among the best conserved ancient Stupas of
central India.

Created with by ForumIAS.com – The Knowledge Network for Civil Services.


Visit http//forumias.com New! http://forumiasacademy – Prelims & Mains Test Series

ForumIAS
PRELIMS MARATHON COMPILATION FOR THE MONTH OF MARCH, 2018

13. Statements 1 and 3 are correct. Carnatic Sangeet has developed in the south Indian states of Tamil
Nadu, Kerala, Andhra Pradesh and Karnataka. These states are known for their strong presentation of
Dravidian culture. Purandardas (1480-1564) is considered to be the father of Carnatic music. It has only
one particular prescribed style of singing.
Statement 2 is incorrect. Tabla is associated with Hindustani music. The principal instruments played
with Carnatic music are Veena, Mrindagum and Mandolin.
Amir Khusro is credited with fashioning the Tabla as a split version of the traditional Indian drum, the
pakhawaj.

14. The string puppets of Karnataka are called Gombeyatta.


They are styled and designed like the characters of Yakshagana, the traditional theatre form of the region.
The puppets have rounded figures with legs, and joints at shoulders, elbows, hips and knees. The
Gombeyatta puppet figures are highly stylized.

15. All of the statements are correct about the initiatives were undertaken by Gandhiji during his stay in
Africa.
During the moderate phase of the struggle (1894-1906) Gandhi set up the Natal Indian Congress and
started a paper Indian opinion. The second phase of the struggle which began in 1906, was characterized
by the use of the method of passive resistance or civil disobedience, which Gandhi named Satyagraha.
It was first used when the Government enacted legislation making it compulsory for Indians to take out
certificates of registration which held their fingerprints. The Indians under Gandhi's leadership decided
not to submit to this discriminatory measure.
The campaign was widened to include protest against a new legislation imposing restrictions on Indian
migration. The poll tax of three pounds imposed on ex-indentured Indians and invalidation of Indian
marriages further widened the campaign.

16. The caves excavated at Ellora are associated with three different religious creeds, viz., Buddhism,
Brahmanism and Jainism. The caves are datable from circa 6th - 7th century A.D. to 11th - 12th century
A.D. In total, there are nearly 100 caves in the hill range out of which 34 caves are famous and visited by
many tourists, out of which Caves 1 to 12 are Buddhist; Caves 13 to 29 are Brahmanical and Caves 30 to
34 are Jaina. Two more groups of caves are noticed on the Elaganga and on an upper terrace, namely, the
Ganesh Leni and Jogeshwari Leni.

17. First Lucknow session: Romesh Chunder Dutt presided over the first Lucknow session held in 1899.
He was a civil servant, writer, translator of Mahabharata and Ramayana, and an economic historian.

18. Statement 1 is correct. The early nationalist did not want a direct struggle. The nature of early
nationalists was constitutional struggle within established constitutional framework. The political
demands were moderate in nature, with absence of mass mobilization. Early nationalists were fully
aware of the fact that India had just entered the process of becoming a nation. Therefore, they wanted to
create a national unity.
Statement 2 is incorrect. Direct struggle was not the agenda of the early nationalist.
Created with by ForumIAS.com – The Knowledge Network for Civil Services.
Visit http//forumias.com New! http://forumiasacademy – Prelims & Mains Test Series

ForumIAS
PRELIMS MARATHON COMPILATION FOR THE MONTH OF MARCH, 2018

19. Statement 1 is incorrect. The 'Rajagopalachari formula' wanted to solve the constitutional crisis
occurred due to the failure of the Cripps Mission.
Statement 2 is correct. While the Congress was ready to corporate with the league for the independence
of the Indian Union, the League did not care for independence of the Union. It was only interested in a
separate nation. Jinnah wanted the Congress to accept the two-nation theory.

20. Statement 1 is incorrect. The Wavell Plan wanted an executive council with all Indian members
except the governor-general and the commander-in-chief.
Statement 2 and 3 are correct.

21. The fifth Calcutta session of the Indian National Congress was held in 1906. It was presided over by
Dadabhai Naoroji.

22. All of the factors contributed towards the popular support for Home rule league.
The mass agitation could not be carried out under the leadership of the Indian National Congress, which
had become, under moderate leadership, a passive and inert political organisation with no political work
among the people to its credit. Therefore, two Home Rule Leagues were started in 1915-16, under the
leadership of Annie Besant and Lokamanya Tilak.
World war involving mutual struggle between the imperial powers destroyed the myth of racial
superiority. The war led to increased misery among the poorer classes of Indians. For them the War had
meant heavy taxation and soaring prices of the daily necessities of life. They were getting ready to join
any movement of protest.

23. Statement 1 is correct. From 1763 to 1856 there were more than 40 major rebellions apart from
hundreds of minor ones. These rebellions were, however, local in character and effects. They were
isolated from each other because each rebellion had a different motive.
Statement 2 is incorrect. All the resistance movements of zamindars and rulers, and tribal chiefs were
violent in nature.
Statement 3 is correct. The forces behind such these popular resistance movements aimed at restoration
of old structures and relations which had been done away with by the British. Each social group had its
own reasons to raise its voice against the colonial powers.
For example, displaced zamindars and rulers wanted to regain their land and estates.
Similarly, the tribal groups rebelled because they did not want the traders and moneylenders to interfere
in their lives.

24. Statement 1 is incorrect. The act abolished dyarchy in provinces and autonomy was given to them.
Dyarchy was introduced at the centre.
Statement 2 is correct. All India federation with two governments i.e. Union and state was created.
Statement 3 is correct. The joining of the princely states in the proposed Indian federation was voluntary

Created with by ForumIAS.com – The Knowledge Network for Civil Services.


Visit http//forumias.com New! http://forumiasacademy – Prelims & Mains Test Series

ForumIAS
PRELIMS MARATHON COMPILATION FOR THE MONTH OF MARCH, 2018

and as a result of the federation did not come into existence.

25. The correct sequence of the events is:


1. The August offer - 1940
2. The Quit India Movement - August, 1942
3. The INA Trial – 1945
4. The Royal Indian Naval Ratings Revolt – February 1946

26. Statement 1 is incorrect. Pre-censorship of Indian newspapers was introduced through the
Censorship of Press Act, 1799 by Lord Wellesley. It was done to anticipate French invasion of India. The
restriction was removed in 1818 by Lord Hastings.
Statement 2 is correct. The new Press Act (1835) required a printer/publisher to give a precise account of
premises of a publication and cease functioning, if required by a similar declaration. Earlier, newspapers
and books can be published without mentioning the place of publication which encouraged anti-
government discussions. Through this act, the government wanted to regulate the printing presses and
newspapers, preservation of copies of books printed in British India.
Statement 3 is correct. Under the Vernacular Press Act of 1878, the printer and publisher could be
required to deposit security which could be forfeited if the regulation were contravened, and press
equipment could be seized if the offence re-occurred.
The Indian Press Act of 1910 empowered the local government to demand a security at registration from
the printer/publisher and forfeit/deregister if it was an offending newspaper, and the printer of a
newspaper was required to submit two copies of each issue to local government free of charge.

27. Both the statements are incorrect.


The Vedic texts constitute of religious literature whereas Sangam texts are secular in nature. Sangam texts
provide information of some towns such as kanchi, korkai, madurai, puhar. They mention of Greek
roman trade. The texts of sangam era provide information on code of conduct for kings and his court, for
other social groups.

28. The Bengali language has not been declared as ‘Classical Languages’ by the Government of India.
Currently there are six classical languages. They are Tamil, Telugu, Kannada, Sanskrit, Malayalam and
Odia.

29. Statement 1 is incorrect. Sarojini Naidu was the lady representative of India at the Second Round
Table Conference held in 1931.
Statement 2 is incorrect. Indian National congress did not participate the First Round Table Conference
held in 1930. But Muslim League participated in it and the representatives were Muhammad Ali Jinnah,
Muhammad Shafi, Aga Khan, Muhammad Ali, Muhammad Zafrulla Khan, A.K. Fazlul Huq etc.
Statement 3 is correct. Third Round Table Conference was held in London on November 17, 1932. This
was just a nominal conference, Congress refused to attend it (not invited, in fact) and in Britain, the Labor
party also refused to not to attend it. The outcome of the Third Round Table conference was the ” White
Created with by ForumIAS.com – The Knowledge Network for Civil Services.
Visit http//forumias.com New! http://forumiasacademy – Prelims & Mains Test Series

ForumIAS
PRELIMS MARATHON COMPILATION FOR THE MONTH OF MARCH, 2018

Paper” issued by the Government. On the basis of this paper, the Government of India Act 1935 was to be
passed.

Created with by ForumIAS.com – The Knowledge Network for Civil Services.


Visit http//forumias.com New! http://forumiasacademy – Prelims & Mains Test Series

ForumIAS
PRELIMS MARATHON COMPILATION FOR THE MONTH OF MARCH, 2018

Economy
Q.1) Consider the following goods:
1. Computers
2. Cars
3. Compressed Natural Gas
4. Petrol
Which of the above goods are durable consumer goods?
a) 1, 2 and 3
b) 3 and 4 only
c) 1 and 2 only
d) All of the above

Q.2) In India, Spot Exchanges refer to electronic trading platforms. Spot Exchanges facilitate purchase and
sale of specified commodities, including:
1. Agricultural commodities
2. Metals
3. Bullion
Select the correct answer using the code given below.
a) 1, 2 and 3
b) 1 only
c) 2 and 3 only
d) 1 and 3 only

Q.3) Consider the following situations in an economy:


1. Poor economic growth
2. High level of unemployment
3. Low level of inflation
Which of the above is/are associated with stagflation?
a) 3 only
b) 1 and 2 only
c) 1 and 3 only
d) 1, 2 and 3

Q.4) Which of the following statements is/are correct about debentures?


1. It is a debt instrument backed by a specific security.
2.It is used by corporate treasury as a tool to raise medium to long-term funds.
Select the correct answer using the code given below.
a) 1 only
b) 2 only
c) Both 1 and 2
d) Neither 1 nor 2

Created with by ForumIAS.com – The Knowledge Network for Civil Services.


Visit http//forumias.com New! http://forumiasacademy – Prelims & Mains Test Series

ForumIAS
PRELIMS MARATHON COMPILATION FOR THE MONTH OF MARCH, 2018

Q.5) With reference to Free Trade Agreements (FTAs), consider the following statements:
1. It completely eliminate customs tariff and non-tariff barriers on substantial trade between two or more
countries or trading blocs.
2. It does not cover government procurement and competition policy etc.
Which of the statements given above is/are correct?
a) 1 only
b) 2 only
c) Both 1 and 2
d) Neither 1 nor 2

Q.6) Which of the following is/are example(s) non-tariff measures mentioned by the World Trade
Organization?
1. Quotas
2. Import licensing systems
3. Sanitary regulations
4. Prohibitions
Select the correct answer suing the code given below.
a) 1, 2 and 3 only
b) 3 only
c) 2, 3 and 4 only
d) 1, 2, 3 and 4

Q.7) India and MERCOSUR have


a) Free Trade Agreement
b) Preferential Trade Agreement
c) Both a and b
d) None of the above

Q.8) Which of the following is/are objective(s) of Niryat Bandhu Scheme?


1. To provide Advance Procurement/ Replenishment of Precious Metals from Nominated Agencies.
2. To reach out to the new and potential exporters including exporters from Micro, Small & Medium
Enterprises (MSMEs).
3. To mentor potential exporters through orientation program.
4. To provide individual facilitation and counselling sessions to individuals interested in export.
Select the correct answer using the code provided below.
a) All of the above
b) 1 and 2 only
c) 2, 3 and 4 only
d) 1 and 4 only

Q.9) With reference to Real Estate Investment Trusts (REITs), consider the following statements:
1. They are listed in stock exchanges so that investors can buy units in the trust.
2. Currently, single-asset REITs are not allowed to bid to raise funds through debt securities in India.
Which of the above statements is/are correct?
a) 1 only
Created with by ForumIAS.com – The Knowledge Network for Civil Services.
Visit http//forumias.com New! http://forumiasacademy – Prelims & Mains Test Series

ForumIAS
PRELIMS MARATHON COMPILATION FOR THE MONTH OF MARCH, 2018

b) 2 only
c) Both 1 and 2
d) Neither 1 nor 2

Q.10) With reference to the National Investment and Infrastructure Fund (NIIF), consider the following
statements:
1. It was set up in December 2015 by adding additional corpus to the National Investment Fund to
catalyze funding into the country’s infrastructure sector.
2. It has been registered with the Securities and Exchange Board of India.
3. Recently, it has partnered with DP World to create an investment platform for ports and logistics
businesses in India.
Which of the above statements is/are correct?
a) 1 and 2 only
b) 1 only
c) 2 and 3 only
d) 1, 2 and 3

Q.11) Which of the following is correct about Collective Investment Schemes?


1. Investors have the day to day control over the management and operation of such scheme.
2. Collective Investment Schemes are required to register with Reserve Bank of India as they accept
deposits from investors directly.
Select the correct answer using the code given below.
a) 1 only
b) 2 only
c) Both 1 and 2
d) Neither 1 nor 2

Q.12) Which among the following correctly describes the term 'Tax buoyancy'?
a) It is the ratio of gross tax collected to the total public debt of the economy.
b) It is the ratio of growth in tax revenue to growth in Gross Domestic Product.
c) It is the ratio of growth in tax revenue to the growth in population.
d) It is the ratio of gross tax collected by the state to the percentage of Gross Domestic Product.

Q.13) With reference to the Pradhan Mantri Suraksha Bima Yojana, consider the following statements:
1. It aims at creating a universal social security system for all Indians especially the poor and
underprivileged.
2. Anyone who falls in the age-bracket of 18-60 years can avail the benefit of this scheme.
Select the correct answer using the codes given below.
a) 1 only
b) 2 only
c) Both 1 and 2
d) Neither 1 nor 2

Created with by ForumIAS.com – The Knowledge Network for Civil Services.


Visit http//forumias.com New! http://forumiasacademy – Prelims & Mains Test Series

ForumIAS
PRELIMS MARATHON COMPILATION FOR THE MONTH OF MARCH, 2018

Q.14) Which of the following can be appointed as Business Correspondents (BCs), as per the Reserve
Bank of India's guidelines on Business Correspondents (BCs)?
1. Retired government employees
2. Retired bank employees
3. Teachers at government schools
4. Owners of kirana shop
5. Individuals who own Petrol Pumps
Select the correct answer using the code given below.
a) 1, 2, 3, 4 and 5
b) 1, 2 and 3 only
c) 2 and 3 only
d) 1, 2, 4 and 5 only

Q.15) Consider the following statements:


1. Marginal Standing Facility (MSF) allows scheduled commercial banks to borrow from the RBI if the
former doesn’t have the required eligible securities above the SLR limit.
2. Unlike Marginal Standing Facility, Bank rate is applied to all commercial banks and there is no
limitation like 2 percent of their respective Net Demand and Time Liabilities (NDTL).
Which of the statements given above is/are correct?
a) 1 only
b) 2 only
c) Both 1 and 2
d) Neither 1 nor 2

Q.16) Consider the following statements:


1. The World Bank classifies the countries into three categories in terms of their per capita income.
2. Low-income countries are those with a per capita income lower than $1005.
3. Currently, India falls in the list of Low-Income Countries (LICs).
Which of the statements given above is/are correct?
a) 1 and 2 only
b) 2 only
c) 1 and 3 only
d) 1, 2 and 3

Q.17) Which of the following can be the sources of human capital formation?
1. Investment in health sector
2. Investment in education
3. Migration
4. On-the-job-training
Select the correct answer using the code given below.
a) 1, 2, 3 and 4
b) 1, 2 and 3 only
c) 3 and 4 only
d) 1, 2 and 4 only

Created with by ForumIAS.com – The Knowledge Network for Civil Services.


Visit http//forumias.com New! http://forumiasacademy – Prelims & Mains Test Series

ForumIAS
PRELIMS MARATHON COMPILATION FOR THE MONTH OF MARCH, 2018

Q.18) Consider the following differences between Marginal Cost of Funds based Lending Rate (MCLR):
1. It has replaced the Base Rate system of lending.
2. It has brought transparency in the methodology followed by banks for determining interest rates on
advances but made borrowing costlier than earlier.
Which of the statements given above is/are correct?
a) 1 only
b) 2 only
c) Both 1 and 2
d) Neither 1 nor 2

Q.19) In an economy a situation of liquidity trap is characterized by which of the following?


1. Low savings rate
2. Expansionary monetary policy
3. High demand for bonds
Select the correct answer using the code given below.
a) 2 and 3 only
b) 2 only
c) 1 and 2 only
d) 1, 2 and 3

Q.20) Consider the following statements:


1. Gross Value Added (GVA) provides the rupee value for the amount of goods and services produced in
an economy after deducting the cost of inputs and raw materials.
2. While GVA gives a picture of the state of economic activity from the producers’ side or supply side, the
GDP gives the picture from the consumers’ side.
Which of the statement given above is/are incorrect?
a) 1 only
b) 2 only
c) Both 1 and 2
d) Neither 1 nor 2

Q.21) Consider the following types of expenditure:


1. Investment expenditure
2. Consumption expenditure
3. Government expenditure
Which of the above is/are responsible for generating aggregate demand for final goods in an economy?
a) 1 and 3 only
c) 2 only
b) 2 and 3 only
d) 1, 2 and 3

Q.22) Which of the following statements is/are correct regarding Special Safeguard mechanism in WTO?
1. It allows developing countries to raise tariffs temporarily to deal with import surges or price falls.
2. It has been included under the Nairobi Package negotiated under the WTO.
Select the correct answer using the code given below.
Created with by ForumIAS.com – The Knowledge Network for Civil Services.
Visit http//forumias.com New! http://forumiasacademy – Prelims & Mains Test Series

ForumIAS
PRELIMS MARATHON COMPILATION FOR THE MONTH OF MARCH, 2018

a) 1 only
b) 2 only
c) Both 1 and 2
d) Neither 1 nor 2

Q.23) Consider the following statements:


1. While a share capital is the credit to the company, a debenture is an ownership capital.
2. Interest is paid on both debentures and shares.
Which of the statements given above is/are correct?
a) 1 only
b) 2 only
c) Both 1 and 2
d) Neither 1 nor 2

Q.24) With reference to District Mineral Foundation (DMF), consider the following statements:
1. It is a non-profit organization.
2. It is mandatory to set up DMF in all districts of the country affected by mining related operations
under the provisions of Mines and Minerals (Development & Regulation) Amendment Act, (MMDRA)
2015.
Select the correct answer using the code given below.
a) 1 only
b) 2 only
c) Both 1 and 2
d) Neither 1 nor 2

Q.25) Which of the following is/are key characteristics of organic farming?


1. Weed, disease and pest control relying primarily on crop rotations, natural predators, diversity.
2. Conservation of wildlife and natural habitats
3. Encouraging soil biological activity and careful mechanical intervention
4. Nitrogen self-sufficiency through the use of legumes and biological nitrogen fixation
5. Effective recycling of crop residues and livestock manures
Select the correct answer using the codes given below.
a) 1 and 3 only
b) 2, 4 and 5 only
c) 1, 3 and 5 only
d) 1, 2, 3, 4 and 5

Q.26) If the supply of money remains the same when there is an increase in demand for money, there will
be
a) a fall in the level of prices
b) an increase in the level of income
c) an increase in the rate of interest
d) a decrease in the rate of interest

Created with by ForumIAS.com – The Knowledge Network for Civil Services.


Visit http//forumias.com New! http://forumiasacademy – Prelims & Mains Test Series

ForumIAS
PRELIMS MARATHON COMPILATION FOR THE MONTH OF MARCH, 2018

Q.27) Consider the following statements:


1. Structural unemployment may occur in two wheeler industry because people are buying more cars.
2. Disguised Unemployment is a situation when additional employment of worker in to a work may not
add anything to total output.
Which of the statements given above is /are correct?
a) 1 only
b) 2 only
c) Both 1 and 2
d) Neither 1 nor 2

Q.28) Which among the following is/are part of Public Debt incurred by the Union government?
1. Public Account Liabilities
2. Internal Debt
3. External Debt
Select the correct answer using the code given below.
a) 1, 2 and 3
b) 1 and 2 only
c) 1 and 3 only
d) 2 and 3 only

Answer Key

Qs. Ans. Qs. Ans. Qs. Ans.


1 C 11 D 21 D
2 A 12 B 22 C
3 B 13 A 23 D
4 B 14 D 24 C
5 D 15 C 25 D
6 D 16 B 26 C
7 B 17 A 27 C
8 C 18 A 28 D
9 A 19 B
10 C 20 D

Explanation
1. Durable goods are a category of consumer products that do not need to be purchased frequently
because they are made to last for a long time (usually lasting for three years or more). They are also called
consumer durables or durables. Examples of durable goods include automobiles, appliances, furniture,
jewelry, consumer electronics and sporting goods. TV, cars etc.
When a washing machine is a durable good, the detergent used in the washing machine on the other
hand is a non-durable good. Other examples are Petrol, Compressed Natural Gas etc.
Created with by ForumIAS.com – The Knowledge Network for Civil Services.
Visit http//forumias.com New! http://forumiasacademy – Prelims & Mains Test Series

ForumIAS
PRELIMS MARATHON COMPILATION FOR THE MONTH OF MARCH, 2018

2. In India, Spot Exchanges refer to electronic trading platforms which facilitate purchase and sale of
specified commodities, including agricultural commodities, metals and bullion by providing spot
delivery contracts in these commodities.

3. Stagflation a condition of slow economic growth and relatively high unemployment – economic
stagnation – accompanied by rising prices, or inflation, or inflation and a decline in Gross Domestic
Product (GDP).

4. Statement 1 is incorrect. A debenture is a debt instrument which is not backed by any specific security;
instead the credit of the company issuing the same is the underlying security.
Statement 2 is correct. Corporate treasury use this as a tool to raise medium- to long-term funds. The
funds raised become part of the capital structure but not share capital of the company.

5. Both the statements are incorrect. FTAs are arrangements between two or more countries or trading
blocs that primarily agree to reduce or eliminate customs tariff and non-tariff barriers on substantial trade
between them. FTAs, normally cover trade in goods (such as agricultural or industrial products) or trade
in services (such as banking, construction, trading etc.). FTAs can also cover other areas such as
intellectual property rights (IPRs), investment, government procurement and competition policy, etc.

6. All are examples of Non-tariff measures.

7. India and MERCOSUR have Preferential Trade Agreement. MERCOSUR is a trading bloc in Latin
America comprising Brazil, Argentina, Uruguay and Paraguay. MERCOSUR was formed in 1991 with the
objective of facilitating the free movement of goods, services, capital and people among the four member
countries.
As a follow up to the Framework Agreement, a Preferential Trade Agreement (PTA) was signed in New
Delhi on January 25, 2004.
A PTA is a limited free trade agreement where partner countries reduce import duties on a few identified
products for the other. While the PTA between India and Mercosur is presently limited to just 450
products, the two sides have raised their ambitions manifold and are now aiming at providing
preferential access to about 3,000 items.

8. Statements 2, 3 and 4 are correct.


The Government has implemented the Niryat Bandhu Scheme with an objective to reach out to the new
and potential exporters including exporters from Micro, Small & Medium Enterprises (MSMEs) and
mentor them through orientation programmes, counselling sessions, individual facilitation, etc., on
various aspects of foreign trade to enable them to get into international trade and boost exports from
India.
Statement 1 is incorrect. Under Export Promotion Capital Goods (EPCG) Scheme and Duty
Exemption/Remission Schemes, Advance Authorisation (AA), Duty Free Import Authorisation (DFIA),
Duty Drawback (DBK), Advance Procurement/ Replenishment of Precious Metals from Nominated
Agencies, Replenishment Authorisation for Gems, Replenishment Authorisation for Consumables and
Advance Authorisation for precious metals are also available for MSME products.

Created with by ForumIAS.com – The Knowledge Network for Civil Services.


Visit http//forumias.com New! http://forumiasacademy – Prelims & Mains Test Series

ForumIAS
PRELIMS MARATHON COMPILATION FOR THE MONTH OF MARCH, 2018

9. Statement 1 is correct. REITs are similar to mutual funds. While mutual funds provide for an
opportunity to invest in equity stocks, REITs allow one to invest in income-generating real estate assets.
REITs raise funds from a large number of investors and directly invest that sum in income-generating
real estate properties (which could be offices, residential apartments, shopping centres, hotels and
warehouses).
The trusts are listed in stock exchanges so that investors can buy units in the trust. REITs are structured as
trusts. Thus, the assets of an REIT are held by an independent trustee on behalf of unit holders.
Statement 2 is incorrect. The Securities and Exchange Board of India (Sebi) on 19 Sep, 2017 amended
norms governing real estate investments trusts (REITs) and infrastructure investment trusts (InvITs),
allowing them to raise funds through debt securities and also permitting single-asset REITs in a bid to
boost the financial instruments.

10. Statement 1 is incorrect. National Investment and Infrastructure Fund (NIIF) is a fund created by the
Government of India for enhancing infrastructure financing in the country.
This is different from the National Investment Fund. Its creation was announced in the Union Budget
2015-16.
The operational framework was approved on 20 August 2015.
Statement 2 is correct. NIIF got registered with SEBI as Category II Alternative Investment Fund (AIF) on
December 28, 2015.
Statement 3 is correct. The National Investment and Infrastructure Fund (NIIF) makes its First
Investment; NIIF has partnered with DP World to create an investment platform for ports, terminals,
transportation and logistics businesses in India. DP World is the world’s fourth largest port operator.

11. A Collective investment scheme is any scheme or arrangement, which satisfies the conditions, referred
to in sub-section (2) of section 11AA of the SEBI Act. Any scheme or arrangement made or offered by any
company under which the contributions, or payments made by the investors, are pooled and utilised
with a view to receive profits, income, produce or property, and is managed on behalf of the investors is a
CIS.
Investors do not have day to day control over the management and operation of such scheme or
arrangement. Hence, statement 1 is incorrect.
Statement 2 is incorrect. A Collective Investment Management Company is a company incorporated
under the provisions of the Companies Act, 1956 and registered with SEBI under the SEBI (Collective
Investment Schemes) Regulations,
1999, whose object is to organize, operate and manage a Collective Investment Scheme.

12. Tax buoyancy is an indicator to measure efficiency and responsiveness of revenue mobilization in
response to growth in the Gross domestic product or National income.
A tax is said to be buoyant if the tax revenues increase more than proportionately in response to a rise in
national income or output.

13. Statement 1 is correct. The Pradhan Mantri Suraksha Bima Yojana is a one year cover Personal
Accident Insurance Scheme, renewable from year to year, offering protection against death or disability
due to accident.
Statement 2 is incorrect. All individual (single or joint) bank account holders in the age 18 to 70 years in
participating banks will be entitled to join in Pradhan Mantri Suraksha Bima Yojana.
Created with by ForumIAS.com – The Knowledge Network for Civil Services.
Visit http//forumias.com New! http://forumiasacademy – Prelims & Mains Test Series

ForumIAS
PRELIMS MARATHON COMPILATION FOR THE MONTH OF MARCH, 2018

14. All of the above expect working teachers at government schools (not retired) can be appointed as
Business Correspondents (BCs).
The scheduled commercial banks including Regional Rural Banks (RRBs) and Local Area Banks (LABs)
may engage Business Correspondents (BCs) subject to compliance with the following guidelines issued
the Reserve Bank of India.

15. Statement 1 is correct. Marginal Standing Facility is a liquidity support arrangement provided by RBI
to scheduled commercial banks if the latter doesn’t have the required eligible securities above the SLR
limit.
Statement 2 is correct. Bank rate is a higher rate, (1% higher than REPO rate) charged by RBI when it
gives loans to commercial banks. Bank rate is different from MSF in the nature that Bank rate is long-
term, applies for all commercial banks and there is no limitation like 2 percent of their respective Net
Demand and Time Liabilities (NDTL).

16. Statement 1 is incorrect. The World Bank is classifying countries in terms of per capita income, into
four categories i.e. Low-Income Countries (LICs), Lower Middle-Income Countries (LMICs), Upper
Middle-Income Countries (UMICs) and High-Income Countries (HICs).
Statement 2 is correct. Low-income countries are those with a per capita income lower than $1005.
Statement 3 is incorrect. India has entered the middle-income position in 2008 and the per capita income
is increasing slowly. The Per Capita Income of the country as per the World Bank’s 2016 estimate is $
1670.

17. All of the above can be the sources of human capital formation.
Human capital refers to stock of ‘skill and expertise’ embodied in humans. Human capital is as important
as physical capital for economic development. Human capital formation is the process of adding to stock
of human capital over time. Human capital can be developed through creation of skilled, trained and
efficient labour force by providing better education, health care facilities, etc.
Highly skilled people can create new ideas and methods of production. Thus, expenditure on education,
on health and on on-job-training are key instruments of human capital formation. Expenditure on
education is one of the most important way of enhancing and enlarging a productive workforce in the
country. Expenditure on health can create more efficient and more productive human capital. Further,
on-the-job-training helps workers to update skills. Training enhances the productivity and is expected to
accelerate the process of human capital formation.

18. Statement 1 is correct. The MCLR methodology for fixing interest rates for advances was introduced
by the Reserve Bank of India with effect from April 1, 2016. This new methodology replaces the base rate
system introduced in July 2010.
MCLR aims to bring transparency in the methodology followed by banks for determining interest rates
on advances. It also aims to ensure availability of bank credit at interest rates which are fair to borrowers
as well as banks. Thus, statement 2 is incorrect.
Other aims behind the introduction of MCLR are:
1. To improve the transmission of policy rates into the lending rates of banks.
2.To enable banks to become more competitive and enhance their long-run value and contribution to
economic growth.
Created with by ForumIAS.com – The Knowledge Network for Civil Services.
Visit http//forumias.com New! http://forumiasacademy – Prelims & Mains Test Series

ForumIAS
PRELIMS MARATHON COMPILATION FOR THE MONTH OF MARCH, 2018

19. The liquidity trap is the situation in which prevailing interest rates are low and savings rates are high,
making monetary policy ineffective. In a liquidity trap, consumers choose to avoid bonds and keep their
funds in savings because of the prevailing belief that interest rates will soon rise.
Because bonds have an inverse relationship to interest rates, many consumers do not want to hold an
asset with a price that is expected to decline.

20. Both the statements are correct. Gross value added is a measure of total output and income in the
economy. It provides the rupee value for the amount of goods and services produced in an economy after
deducting the cost of inputs and raw materials that have gone into the production of those goods and
services. It also gives sector-specific picture like what is the growth in an area, industry or sector of an
economy.
While GVA gives a picture of the state of economic activity from the producers’ side or supply side, the
GDP gives the picture from the consumers’ side or demand perspective. Both measures need not match
because of the difference in treatment of net taxes. This is one of the reasons that in the first quarter of
2015, GDP growth was stronger at 7.5%, while GVA growth was 6.1%.

21. Aggregate demand refers to the total demand for final goods and services in an economy. Aggregate
demand is measured by the total expenditure of the community on goods and services and consists of ex-
ante consumption, ex-ante investment, government spending etc. Thus, all are responsible for generating
aggregate demand for final goods in an economy.

22. Both the statements are correct.


WTO's Special Safeguard Mechanism (SSM) is a protection measure allowed for developing countries to
take contingency restrictions against agricultural imports that are causing injuries to domestic farmers. It
is a tool that will allow developing countries to raise tariffs temporarily to deal with import surges or
price falls.
The “Nairobi Package” was adopted at the WTO's Tenth Ministerial Conference, held in Nairobi, Kenya,
from 15 to 19 December 2015. It contains a series of six Ministerial Decisions on agriculture, cotton and
issues related to least-developed countries (LDCs). A Ministerial Declaration outlining the Package and
the future work of the WTO was adopted at the end of the five-day Conference. The matters related to
agriculture mentioned in the Nairobi Package are Special Safeguard Mechanism for Developing Country
Members, Public Stockholding for Food Security Purposes, Export Competition.

23. Statement 1 is incorrect. While share represents part of ownership of a company, debenture
acknowledges loan or debt to the company. Thus, a shareholder is a participant in the profits as well as
losses of the company.
Statement 2 is incorrect. Dividend is paid on share, which is an appropriation of profits, but a debenture
holder is paid interest over the lifetime of the debenture and principal amount at the end of life.

24. Both the statements are correct.


District Mineral Foundation (DMF) is a trust set up as a non-profit body, in those districts affected by the
mining works, to work for the interest and benefit of persons and areas affected by mining related
operations. It is funded through the contributions from miners. Its manner of operation comes under the
jurisdiction of the relevant State Government.
Created with by ForumIAS.com – The Knowledge Network for Civil Services.
Visit http//forumias.com New! http://forumiasacademy – Prelims & Mains Test Series

ForumIAS
PRELIMS MARATHON COMPILATION FOR THE MONTH OF MARCH, 2018

Setting up of District Mineral Foundations (DMFs) in all districts in the country affected by mining
related operations was mandated through the Mines and Minerals (Development & Regulation)
Amendment Act, (MMDRA) 2015. On 16 September 2015, Central Government issued a notification
directing states to set up DMF.

25. All are key characteristics of organic farming.


1. Protecting the long term fertility of soils by maintaining organic matter levels, encouraging soil
biological activity, and careful mechanical intervention
2. Providing crop nutrients indirectly using relatively insoluble nutrient sources which are made
available to the plant by the action of soil micro-organisms
3. Nitrogen self-sufficiency through the use of legumes and biological nitrogen fixation, as well as
effective recycling of organic materials including crop residues and livestock manures
4. Weed, disease and pest control relying primarily on crop rotations, natural predators, diversity,
organic manuring, resistant varieties and limited (preferably minimal) thermal, biological and
chemical intervention
5. The extensive management of livestock, paying full regard to their evolutionary adaptations,
behavioural needs and animal welfare issues with respect to nutrition, housing, health, breeding
and rearing
6. Careful attention to the impact of the farming system on the wider environment and the
conservation of wildlife and natural habitats

26. If the supply of money remains the same when there is an increase in demand for money, there will be
an increase in the rate of interest.
An increase in the supply of money typically lowers interest rates, which in turns generates more
investment and puts more money in the hands of consumers, thereby stimulating spending. Businesses
respond by ordering more raw materials and increasing production.

27. Both the statements are correct.


Structural unemployment refers to a mismatch between the jobs available and the skill levels of the
unemployed. It is caused by forces other than the business cycle. It occurs when an underlying shift in the
economy makes it difficult for some groups to find jobs.
Disguised unemployment exists where part of the labor force is either left without work or is working in
a redundant manner where worker productivity is essentially zero. It is unemployment that does not
affect aggregate output.

28. Public Debt in India includes only Internal and External Debt incurred by the Central Government.
Internal Debt includes liabilities incurred by resident units in the Indian economy to other resident units,
while External Debt includes liabilities incurred by residents to non-residents. The Public Account
Liabilities is a type of liability.
The total Union Government Liabilities constitutes the following three categories:
1. Internal Debt
2. External Debt
3. Public Account Liabilities

Created with by ForumIAS.com – The Knowledge Network for Civil Services.


Visit http//forumias.com New! http://forumiasacademy – Prelims & Mains Test Series

ForumIAS
PRELIMS MARATHON COMPILATION FOR THE MONTH OF MARCH, 2018

Polity
Q.1) Consider the following statements:
1. The Lok Sabha represents the people of India as a whole.
2. The election to Lok Sabha is based on the principle of universal adult franchise.
Which of the statements given above is /are correct?
a) 1 only
b) 2 only
c) Both 1 and 2
d) Neither 1 nor 2

Q.2) In India, the first Municipal Corporation was set up in which one among the following?
a) Calcutta
b) Bombay
c) Madras
d) Delhi

Q.3) With reference to sixth schedule, which of the following statements is/are correct?
1. Autonomous districts created under sixth schedule fall outside the executive authority of state.
2. The district and regional councils in the sixth schedule areas are empowered to assess and collect land
revenue.
Select the correct answer using the code given below.
a) 1 only
b) 2 only
c) Both 1 and 2
d) Neither 1 nor 2

Q.4) With reference to Delimitation Commission of India, consider the following statements:
1. Delimitation Commission also known as Delimitation Commission or a Boundary Commission.
2. Order of the Delimitation Commission can be challenged only in the Supreme Court.
Which of the above statements is/are correct?
a) 1 only
b) 2 only
c) Both 1 and 2
d) Neither 1 nor 2

Q.5) Consider the following statements regarding National Human Rights Commission (NHRC):
1. It is a Constitutional body.
2. It can only look into matter of Human rights violation within 1 year of its occurrence.
Which of the statements given above is/are correct?
Created with by ForumIAS.com – The Knowledge Network for Civil Services.
Visit http//forumias.com New! http://forumiasacademy – Prelims & Mains Test Series

ForumIAS
PRELIMS MARATHON COMPILATION FOR THE MONTH OF MARCH, 2018

a) 1 only
b) 2 only
c) Both 1 and 2
d) Neither 1 nor 2

Q.6) Consider the following statements:


The Parliamentary Committee on Public Accounts
1. has 17 members from the Lok Sabha.
2. examines the annual audit reports of the Comptroller and Auditor General of India (CAG).
3. scrutinizes appropriation accounts and the finance accounts of the Union government and any other
accounts laid before the Lok Sabha.
Which of the statements given above is / are correct?
a) 1 only
b) 2 and 3 only
e) 3 only
d) 1, 2 and 3

Q.7) Which among the following statement is incorrect about the Comptroller and Auditor General
(CAG) of India?
a) He controls the entire financial system of the country at both the levels—the Centre and the state.
b) He swears to devote himself to the service and well-being of the people of India.
c) He can be removed by the President on same grounds and in the same manner as a judge of the
Supreme Court.
d) He is not eligible for further office.

Q.8) Consider the following statements:


1. The Constitution provides a uniform administrative system to all union territories.
2. The Constitution allows the Union Government to administer an acquired territory as a union territory.
Which of the above statements is/are incorrect?
a) 1 only
b) 2 only
c) Both 1 and 2
d) Neither 1 nor 2

Q.9) Consider the following statements:


A Tribes Advisory Council
1. has the status of a constitutional body.
2. can be establish also in any State having Scheduled Tribes but not Scheduled Areas therein.
Which of the above statements is/are correct?
a) 1 only
b) 2 only
c) Both 1 and 2
Created with by ForumIAS.com – The Knowledge Network for Civil Services.
Visit http//forumias.com New! http://forumiasacademy – Prelims & Mains Test Series

ForumIAS
PRELIMS MARATHON COMPILATION FOR THE MONTH OF MARCH, 2018

d) Neither 1 nor 2

Q.10) The Members of State Human Rights Commission can be removed in which of the following
manner?
a) By the Governor after referring the matter to the High Court for an inquiry.
b) By the President after referring the matter to the Supreme Court for an inquiry.
b) By the President after consulting with the Governor and Chief Minister of the state.
d) By the Governor after referring the matter to the Supreme Court for an inquiry.

Q.11) Consider the following statements:


1. The recommendations of State Human Rights Commission are binding on the state government or
authority.
2. The State Human Rights Commission should be informed by the state government about the action
taken on the former's recommendations within one month.
Which of the above statements is/are correct?
a) 1 only
b) 2 only
c) Both 1 and 2
d) Neither 1 nor 2

Q.12) Consider the following statements:


1. The Indian Constitution was the first constitution in world to begin with a Preamble.
2. The Preamble is a part of the Constitution and gives clarity in interpretation of the Constitution.
3. The Preamble states that the Constitution derives its authority from the constituent assembly of India.
Which of the above statements is/are correct?
a) 3 only
b) 1 and 3 only
c) 2 only
d) 1, 2 and 3

Q.13) Which of the following are part of the Committee formed to recommend on the appointment of
Information Commissioners?
1. The Chairman of the Rajya Sabha
2. The Leader of Opposition in the Lok Sabha
3. The Prime Minister
4. The Chairman of Lok Sabha
5. A Union Cabinet Minister nominated by the Prime Minister
Select the correct answer using the code given below.
a) 1, 4 and 5 only
b) 2, 3 and 5 only
c) 1, 2, 3 and 4 only
d) All of the above
Created with by ForumIAS.com – The Knowledge Network for Civil Services.
Visit http//forumias.com New! http://forumiasacademy – Prelims & Mains Test Series

ForumIAS
PRELIMS MARATHON COMPILATION FOR THE MONTH OF MARCH, 2018

Q.14) Consider the following statements:


1. The State Legislative Assembly alone can decide in matters like enlargement of the jurisdiction of the
state public service commission.
2. The ministers of state government are equally responsible to both the Legislative Assembly and the
Legislative Council.
Which of the above statements is/are correct?
a) 1 only
b) 2 only
c) Both 1 and 2
d) Neither 1 nor 2

Q.15) Which of the following statements is/are correct about the Finance Commission?
1. It is a quasi-judicial body.
2. It can be constituted by the President even earlier than the expiration of every five years.
3. It consists of a chairman and two other members.
Select the correct answer using the code given below.
a) 1 only
b) 2 and 3 only
c) 1 and 2 only
d) 1, 2 and 3

Q.16) According to 73rd Constitutional Amendment Act, the chairperson of the Panchayat at the village
level is elected
a) Directly elected by the people.
d) Indirectly from amongst the elected members of Panchayat.
b) Through nominated by the Gram Sabha from amongst its members.
d) As per the manner as the state legislature determines

Q.17) Consider the following statements:


The President is elected by members of Electoral College consisting of:
1. The elected members of Rajya Sabha.
2. The elected and nominated members of Lok Sabha.
3. The elected members of the legislative assemblies of the states.
4. The elected members of the legislative assemblies of the Union Territories of Delhi and Puducherry
Which of the above statements is/are correct?
a) All of the above
b) 1, 2 and 3 only
c) 1, 3 and 4 only
d) 2, 3 and 4 only

Created with by ForumIAS.com – The Knowledge Network for Civil Services.


Visit http//forumias.com New! http://forumiasacademy – Prelims & Mains Test Series

ForumIAS
PRELIMS MARATHON COMPILATION FOR THE MONTH OF MARCH, 2018

Q.18) With reference to the President of India, consider the following statements:
1. The President must act according to aid and advice of the Council of Ministers.
2. The President can reject an advice of Council of Ministers sent after reconsideration.
3. The President acts according to his own discretion when the Council of Ministers headed by the Prime
Minister resigns.
Which of the above statements is/are correct?
a) 1 and 2 only
b) 1 only
c) 2 and 3 only
d) 1, 2 and 3

Q.19) The Central law prevails over the state law, if


1. In case of a conflict between the central and state law on a subject enumerated in the Concurrent List.
2. If a state law fails to get the President’s assent.
3. In case of overlapping between the Union List and the Concurrent List.
Which of the statements given above is/are correct?
a) 2 only
b) 1 and 3 only
c) 1 and 2 only
d) 1, 2 and 3

Q.20) Consider the following statements about the election of the members of Rajya Sabha:
1. The fourth Schedule of the Constitution fixes the number of members to be elected from each State.
2. All the members of the State Legislative Assembly participate in the election.
3. Election for the vacant seats of Rajya Sabha are held every third year.
Which of the statements given above is/are correct?
a) 1 only
b) 2 and 3 only
c) 1 and 3 only
d) 1, 2 and 3

Q.21) Consider the following statements:


1. The Parliament cannot determine the qualifications of members of the Finance Commission.
2. A person qualified to be appointed as judge of high court can be selected as a member of the Finance
Commission.
Which of the statements given above is/are correct?
a) 1 only
b) 2 only
c) Both 1 and 2
d) Neither 1 nor 2

Created with by ForumIAS.com – The Knowledge Network for Civil Services.


Visit http//forumias.com New! http://forumiasacademy – Prelims & Mains Test Series

ForumIAS
PRELIMS MARATHON COMPILATION FOR THE MONTH OF MARCH, 2018

Q.22) Which of the following is/are not a Socialist Principles of directive Principles of state Policy?
1. To raise the level of nutrition and the standard of living of people and to improve public health.
2. To secure a living wage, a decent standard of life and social and cultural opportunities for all workers.
3. To secure for all citizens a uniform civil code throughout the country.
Select the correct answer by using the code given below.
a) 3 only
b) 1 and 2
c) 2 and 3
d) 1, 2 and 3

Q.23) Which of the following words were added to the Preamble of the Indian Constitution through the
42nd Amendment Act?
1. Socialist
2. Secular
3. Equality
4. Integrity
Select the correct answer using the code given below.
a) 2 and 4 only
b) 1 and 4 only
c) 1, 2 and 4 only
d) 1, 2 and 3 only

Q.24) Consider the following statements:


1. The Supreme Court exclusively enforces the central law while the high courts enforce state laws.
2. Only the Supreme Court can declare the parliamentary laws as unconstitutional through the power of
judicial review.
Which of the statements given above is/are correct?
a) 1 only
b) 2 only
c) Both 1 and 2
d) Neither 1 nor 2

Q.25) Consider the following statements:


1. Fundamental Duties can be used by the courts to determine the constitutionality of a law.
2. Fundamental Duties can be enforced by the Parliament through legislation.
Which of the above statements is/are incorrect?
a) 1 only
b) 2 only
c) Both 1 and 2
d) Neither 1 nor 2

Created with by ForumIAS.com – The Knowledge Network for Civil Services.


Visit http//forumias.com New! http://forumiasacademy – Prelims & Mains Test Series

ForumIAS
PRELIMS MARATHON COMPILATION FOR THE MONTH OF MARCH, 2018

Q.26) With reference to Indian Constitution, consider the following statements:


1. Formation of new states requires amendment in Constitution under article 368.
2. Amendment of the provisions related to federal features of the constitution requires special majority
and consent of the half of the states' legislatures.
Which of the statement given above is/are correct?
a) 1 only
b) 2 only
c) Both 1 and 2
d) Neither 1 nor 2

Q.27) The term ‘Grass root democracy’ is used to refer


a) Lok Pal
b) Multi Party political system
c) Panchayati Raj
d) Universal Adult Franchise

Q.28) With reference to the Parliament of India, Consider the following statements:
1. The Rajya Sabha alone can initiate the removal of the Vice-President.
2. The Rajya Sabha cannot discuss the Annual Financial Statement.
3. No-confidence motion can be passed in either houses of the Parliament.
Which of the statements given above is/are correct?
a) 1 only
b) 2 and 3 only
c) 1 and 3 only
d) 1, 2 and 3

Answer Key

Qs. Ans. Qs. Ans. Qs. Ans.


1 C 11 B 21 B
2 C 12 C 22 A
3 B 13 B 23 C
4 A 14 D 24 D
5 B 15 C 25 D
6 B 16 D 26 B
7 B 17 C 27 C
8 A 18 B 28 A
9 C 19 D
10 B 20 A

Created with by ForumIAS.com – The Knowledge Network for Civil Services.


Visit http//forumias.com New! http://forumiasacademy – Prelims & Mains Test Series

ForumIAS
PRELIMS MARATHON COMPILATION FOR THE MONTH OF MARCH, 2018

Explanation
1. Both the statements are correct. The Rajya Sabha is the Upper House (Second Chamber or House of
Elders) and the Lok Sabha is the Lower House (First Chamber or Popular House). The former represents
the states and union territories of the Indian Union, while the latter represents the people of India as a
whole.
The representatives of states in the Lok Sabha are directly elected by the people from the territorial
constituencies in the states. The election is based on the principle of universal adult franchise. Every
Indian citizen who is above 18 years of age and who is not disqualified under the provisions of the
Constitution or any law is eligible to vote at such election. The voting age was reduced from 21 to 18
years by the 61st Constitutional Amendment Act, 1988.

2. In 1687-88, the first municipal corporation in India was set up at Madras.

3. Statement 1 is incorrect. The tribal areas in the four states of Assam, Meghalaya, Tripura and Mizoram
have been constituted as autonomous districts. But, they do not fall outside the executive authority of the
state concerned.
Statement 2 is correct. The district and regional councils are empowered to assess and collect land
revenue and to impose certain specified taxes.
The district and regional councils administer the areas under their jurisdiction. They can make laws on
certain specified matters like land, forests, canal water, shifting cultivation, village administration,
inheritance of property, marriage and divorce, social customs and so on. But all such laws require the
assent of the Governor.

4. Statement 1 is correct. Delimitation literally means the act or process of fixing limits or boundaries of
territorial constituencies in a country or a province having a legislative body. The job of delimitation is
assigned to a high power body. Such a body is known as Delimitation Commission or a Boundary
Commission.
Statement 2 is incorrect. The Delimitation Commission in India is a high-power body whose orders have
the force of law and cannot be called in question before any court.

5. Statement 1 is incorrect. National Human Rights Commission is a statutory (and not a constitutional)
body. It was established in 1993 under a legislation enacted by the Parliament, namely, the Protection of
Human Rights Act, 1993. This Act was amended in 2006.
Statement 2 is correct. The commission is not empowered to inquire into any matter after the expiry of
one year from the date on which the act constituting violation of human rights is alleged to have been
committed.

6. Statement 1 is incorrect. At present, the Public Accounts Committee consists of 22 members (15 from
the Lok Sabha and 7 from the Rajya Sabha).
Statement 2 is correct. The function of the Public Accounts Committee is to examine the annual audit
reports of the Comptroller and Auditor General of India (CAG), which are laid before the Parliament by
the President.

Created with by ForumIAS.com – The Knowledge Network for Civil Services.


Visit http//forumias.com New! http://forumiasacademy – Prelims & Mains Test Series

ForumIAS
PRELIMS MARATHON COMPILATION FOR THE MONTH OF MARCH, 2018

Statement 3 is correct. The Public Accounts Committee is empowered to examine the appropriation
accounts and the finance accounts of the Union government and any other accounts laid before the Lok
Sabha.

The appropriation accounts compare the actual expenditure with the expenditure sanctioned by the
Parliament through the appropriation act, while the finance accounts shows the annual receipts and
disbursements of the Union government.

7. Statement b is incorrect. The CAG, before taking over his office, makes and subscribes before the
President an oath or affirmation:
1. To bear true faith and allegiance to the Constitution of India;
2. To uphold the sovereignty and integrity of India;
3. To duly and faithfully and to the best of his ability, knowledge and judgement perform the duties of his
office without fear or favour, affection or ill-will; and
4. To uphold the Constitution and the laws.

8. Statement 1 is incorrect. Articles 239 to 241 in Part VIII of the Constitution deal with the union
territories. Even though all the union territories belong to one category, there is no uniformity in their
administrative system.
Statement 2 is correct. The Constitution does not contain any separate provisions for the administration
of acquired territories. But, the constitutional provisions for the administration of union territories also
apply to the acquired territories.

9. Statement 1 is correct. Under Fifth Schedule, the most important institution is the Tribes Advisory
Council. In the Tribes Advisory Council, Scheduled Tribe MLA’s in the state consists of its three-fourth
membership. Since Tribes Advisory Council is formed by the provisions of the Fifth Schedule it has the
status of a constitutional body. The Schedule is aimed towards making the State responsible towards the
promotion of educational and economic interests of Tribals.
Statement 2 is correct. With regard to the Tribes Advisory Council, the Para 4 (1) of the Fifth Schedule to
the Constitution envisages that “There shall be established in each State having Scheduled Areas therein
and, if the President so directs, also in any State having Scheduled Tribes but not Scheduled Areas
therein. In terms of clause (2) of Para 4 of the Fifth Schedule, it shall be the duty of the Tribes Advisory
Council to advise on such matters pertaining to the welfare and advancement of the Scheduled Tribes in
the State as may be referred to them by the Governor.

10. The Chairperson and members of a State Human Rights Commission are appointed by the Governor.
They can be removed only by the President (and not by the Governor). The President can remove them
on the same grounds and in the same manner as he can remove the chairperson or a member of the
National Human Rights Commission.
If the Supreme Court, after the inquiry, upholds the cause of removal and advises so, then the President
can remove the chairperson or a member.

11. Statement 2 is incorrect. The State Human Rights Commission's recommendations are not binding on
the state government or authority.
Created with by ForumIAS.com – The Knowledge Network for Civil Services.
Visit http//forumias.com New! http://forumiasacademy – Prelims & Mains Test Series

ForumIAS
PRELIMS MARATHON COMPILATION FOR THE MONTH OF MARCH, 2018

Statement 2 is correct. The State Human Rights Commission should be informed about the action taken
on its recommendations within one month.

12. Statement 1 is incorrect. The American constitution was the first to begin with a Preamble. India
adopted this practice.
Statement 2 is correct. In Keshavananda Bharti case, The Supreme Court of India has held that preamble
is a part of Constitution; and if there is any lack of clarity in the constitution, the judiciary turns to the
Preamble in its interpretation of the relevant provisions.
Statement 3 is incorrect. The Preamble states that the Constitution derives its authority from the people
of India not from the constituent assembly.

13. The Central Information Commission consists of a Chief Information Commissioner and not more
than ten Information Commissioners.
They are appointed by the President on the recommendation of a committee consisting of the Prime
Minister as Chairperson, the Leader of Opposition in the Lok Sabha and a Union Cabinet Minister
nominated by the Prime Minister.
Where the Leader of Opposition in the Lok Sabha has not been recognized as such, the Leader of the
single largest group in opposition of the Government in the Lok Sabha shall be deemed to be the Leader
of the Opposition.

14. Statement 1 is incorrect. Both the Assembly and the Council have equal power in enlargement of the
jurisdiction of the State Public Service Commission.
Statement 2 is incorrect. Ministers including the chief minister can be members of either House of the
state legislature. However, irrespective of their membership, they are responsible only to the assembly.

15. Statement 1 is correct. Article 280 provides for a Finance Commission as a quasi-judicial body.
Statement 2 is correct. The Finance Commission is constituted by the President every fifth year or even
earlier.
Statement 3 is incorrect. The Finance Commission consists of a chairman and four other members to be
appointed by the President.

16. The chairperson of a Panchayat at the village level shall be elected in such manner as the state
legislature determines.

17. The President is elected not directly by the people but by members of Electoral College consisting of:
1. The elected members of both the Houses of Parliament;
2. The elected members of the legislative assemblies of the states; and
3. The elected members of the legislative assemblies of the Union Territories of Delhi and Puducherry.
This provision was added by the 70th Constitutional Amendment Act of 1992 with effect from June 1,
1995.
The nominated members of both of Houses of Parliament, the nominated members of the state legislative
assemblies, the members (both elected and nominated) of the state legislative councils (in case of the
bicameral legislature) and the nominated members of the Legislative Assemblies of Delhi and
Puducherry do not participate in the election of the President. Where an assembly is dissolved, the

Created with by ForumIAS.com – The Knowledge Network for Civil Services.


Visit http//forumias.com New! http://forumiasacademy – Prelims & Mains Test Series

ForumIAS
PRELIMS MARATHON COMPILATION FOR THE MONTH OF MARCH, 2018

members cease to be qualified to vote in presidential election, even if fresh elections to the dissolved
assembly are not held before the presidential election.

18. Statement 1 is correct. Article 74 (1) says that there shall be a Council of Ministers with the Prime
Minister at the head to aid and advise the President who shall in the exercise of his functions, act in
accordance with such advice.
Statement 2 is incorrect. The President may require the Council of Ministers to reconsider such advice.
After such a reconsideration, the President has to accept such an advice. Thus, the President cannot reject
an advice of Council of Ministers.
Statement 3 is incorrect. Whenever a council of minister resigns, it shall continue as caretaker
government until the next council of ministers swears in. Thus, there shall always be a council of minister
to aid and advice the President.

19. All the statements are correct.

The Constitution expressly secure the predominance of the Union List over the State List and the
Concurrent List, and that of the Concurrent List over the State List. Thus, in case of overlapping between
the Union List and the State List, the former should prevail.
In case of overlapping between the Union List and the Concurrent List, it is again the former which
should prevail. Where there is a conflict between the Concurrent List and the State List, it is the former
that should prevail.
In case of a conflict between the Central law and the state law on a subject enumerated in the Concurrent
List, the Central law prevails over the state law. But, if the state law has been reserved for the
consideration of the president and has received his assent, then the state law prevails in that state. But, it
would still be competent for the Parliament to override such a law by subsequently making a law on the
same matter.

20. Statement 1 is correct. The number of members to be elected from each State has been fixed by the
fourth schedule of the Constitution.
Statement 2 is incorrect. Only the elected members of the state assemblies elect the members of the Rajya
Sabha from that state.
Statement 3 is incorrect. Every two years, one third members of the Rajya sabha complete their terms.
Thus, elections are held every second year.

21. Statement 1 is incorrect. The Constitution authorises the Parliament to determine the qualifications of
members of the commission and the manner in which they should be selected.
Statement 2 is correct. Through the Finance Commission Act, 1951; the Parliament has specified the
qualifications of the chairman and members of the commission.The chairman should be a person having
experience in public affairs and the four other members should be selected from amongst the following:
1. A judge of high court or one qualified to be appointed as one.
2. A person who has specialised knowledge of finance and accounts of the government.
3. A person who has wide experience in financial matters and in administration.
4. A person who has special knowledge of economics.

Created with by ForumIAS.com – The Knowledge Network for Civil Services.


Visit http//forumias.com New! http://forumiasacademy – Prelims & Mains Test Series

ForumIAS
PRELIMS MARATHON COMPILATION FOR THE MONTH OF MARCH, 2018

22. The Socialistic Principles of Directive Principles of State Policy reflect the ideology of socialism. To
secure for all citizens a uniform civil code throughout the country represents the ideology of liberalism
not a Socialist Principles.

23. The 42nd Amendment also amended the Preamble and changed the description of India from
"sovereign democratic republic" to a "sovereign, socialist secular democratic republic". It changed the
words "unity of the nation" to "unity and integrity of the nation".

24. Statement 1 is incorrect. India has an integrated judicial system. The Supreme Court stands at the top
of the integrated judicial system in the country. Below it, there are high courts at the state level. Under a
high court, there is a hierarchy of subordinate courts, that is, district courts and other lower courts. This
single system of courts enforces both the central laws as well as the state laws.
Statement 2 is incorrect. Both the Supreme Court and High courts have the power of judicial review on
parliamentary laws. The power of judicial review over legislative action vested in the High Courts under
Article 226 and in the Supreme Court under Article 32 of the Constitution is an integral and essential
feature of the Constitution, constituting part of its basic structure.
However, the power of judicial review vested on the high court is subject to decision of the Supreme
Court. If a High Court declares a law null and void, the Supreme Court has the final say and it can
modify or reject the high court’s decision. Judicial Review can be conducted in respect of all Central and
State laws, executive orders and ordinances.

25. Both the statements are correct. The Supreme Court in 1992 stated that Fundamental duties can be
used in determining the constitutionality of any law.
The Parliament is free to enforce Fundamental duties through a suitable legislation. Many legislations like
the Prevention of Insults to National Honour Act, 1971 make insult to national symbols a punishable act.

26. Statement 1 is incorrect. Formation of new state requires amendment in constitution under article 3
by simple majority and it is not considered as amendment under the article 368.
Statement 2 is correct. Special majority under article 368 required for amendment by parliament is the
majority of the total membership of each House and a majority of two-thirds of the members present and
voting. Amendment of the provisions related to federal features of the constitution requires consent of
the half of the states along with the special majority.

27. The Panchayati Raj system aims to promote democracy in grass root level. The 73rd Amendment of
the Constitution provided a provision for the mandatory creation of the Gram Sabha. As per the act, it
would comprise of all the adult members registered as voters in the Panchayat area. Its role and functions
are decided by State legislation.
Panchayati Raj is the third tier of the government aimed at development and social justice at the local
level and acts as an instrument of decentralization of powers.

28. Statement 1 is correct. The Vice-President can be removed by a resolution of the Rajya Sabha passed
by an absolute majority (i.e., a majority of the total members of the House) and agreed to by the Lok
Sabha. But, no such resolution can be moved unless at least 14 days’ advance notice has been given.
Statement 2 is incorrect. The Rajya Sabha can discuss the Annual Financial Statement or the Budget.

Created with by ForumIAS.com – The Knowledge Network for Civil Services.


Visit http//forumias.com New! http://forumiasacademy – Prelims & Mains Test Series

ForumIAS
PRELIMS MARATHON COMPILATION FOR THE MONTH OF MARCH, 2018

Statement 3 is incorrect. No-confidence motion can be moved only in Lok Sabha (or state assembly as
the case may be). It is not allowed in Rajya Sabha (or state legislative council).

Created with by ForumIAS.com – The Knowledge Network for Civil Services.


Visit http//forumias.com New! http://forumiasacademy – Prelims & Mains Test Series

ForumIAS
PRELIMS MARATHON COMPILATION FOR THE MONTH OF MARCH, 2018

Science and Technology


Q.1) Consider the following statements:
1. Thermonuclear bombs are more powerful than fission bombs.
2. Nuclear fusion is a process in which atomic nuclei are fused together to form heavier nuclei.
Which of the statements given above is/are correct?
a) 1 only
b) 2 only
c) Both 1 and 2
d) Neither 1 nor 2

Q.2) With reference to Graphene, consider the following statements:


1. It is the thinnest compound known till date.
2. It enables electrons to flow much faster than silicon.
Which of the statements given above is/are correct?
a) 1 only
b) 2 only
c) Both 1 and 2
d) Neither 1 nor 2

Q.3) Consider the following pairs:


Industrial Chemicals Their Usage
1. Nitrogen: Used to cool concrete, improving the properties of the building material
2. Ethylene: Used to produce synthetic rubber
3. Chlorine: Used as a rust remover
Which of the pairs given above is/are incorrectly matched?
a) 3 only
b) 1 and 3 only
c) 2 only
d) 1, 2 and 3

Q.4) Which of the following phenomenon occur due to the principle of Total Internal Reflection?
1. A distant patch of road appearing to be wet on a hot summer day.
2. Transmission of data through optical fiber.
3. A pencil immersed in water looks bent.
Select the correct answer using the code given below.
a) 1, 2 and 3 only
b) 2 and 3 only
c) 1 and 2 only
d) 2 only

Created with by ForumIAS.com – The Knowledge Network for Civil Services.


Visit http//forumias.com New! http://forumiasacademy – Prelims & Mains Test Series

ForumIAS
PRELIMS MARATHON COMPILATION FOR THE MONTH OF MARCH, 2018

Q.5) Consider the following statements:


1. A fuel cell produces electricity through Smouldering combustion.
2. Using of Hydrogen fuel cells eliminates greenhouse gases over the whole cycle.
Which of the above statements is/are incorrect?
a) 1 only
b) 2 only
c) Both 1 and 2
d) None of the above

Q.6) Which among the following statements correctly describes the mechanisms of photovoltaic cell?
a) It converts mechanical energy into chemical energy.
b) It converts visible light into direct current.
c) It converts mechanical energy into light energy.
d) It converts chemical energy into electric energy.

Q.7) The power of a lens is measured in


a) Candela
b) Fathom
c) Dioptres
d) None of the above

Q.8) With reference to bioluminescence, consider the following statements:


1. It occurs through a chemical reaction.
2. It can be observed among sharks also.
Which of the above statements is/are correct?
a) 1 only
b) 2 only
c) Both 1 and 2
d) Neither 1 nor 2

Q.9) Consider the following events:


1. Colliding black holes
2. Supernovae or Supernova
3. Coalescing neutron stars or white dwarf stars
Which of the above event(s) can produce gravitational waves?
a) 1, 2 and 3
b) 1 only
b) 1 and 3
c) 2 only

Created with by ForumIAS.com – The Knowledge Network for Civil Services.


Visit http//forumias.com New! http://forumiasacademy – Prelims & Mains Test Series

ForumIAS
PRELIMS MARATHON COMPILATION FOR THE MONTH OF MARCH, 2018

Q.10) Which of the following statements is/are correct about the Hubble Space Telescope?
1. It is the first major optical telescope to be placed in space.
2. It can see space better than telescopes on Earth can.
Select the correct answer using the code given below.
a) 1 only
b) 2 only
c) Both 1 and 2
d) None of the above

Q.11) Consider the following statements:


The Cell Nucleus
1. is found in both organisms prokaryotes and prokaryotes.
2. stores the cell's hereditary material
3. coordinates the cell's activities
Which of the above statements is/are incorrect?
a) 1 only
b) 2 and 3 only
c) 3 only
d) 1, 2 and 3

Q.12) Consider the following statements about excitonium, a new form of matter recently in the news:
1. It is made up of erbium, a rare-earth element.
2. It exhibits macroscopic quantum phenomena, like a superconductor.
Which of the above statements is/are incorrect?
a) 1 only
b) 2 only
c) Both 1 and 2
d) None of the above

Q.13) Which among the following distinguishes exocrine gland from endocrine gland?
a) The way secretion is accomplished
b) Where the secretion ends up
c) How the secretion gets processed
d) The type of secreted substance

Q.14) For which of the following purposes a convex mirror is used?


1. Hallway safety mirrors
2. Shaving mirror
3. Driveways
4. Automated teller machines
Select the correct answer using the code given below:
a) 1, 2, 3 and 4
Created with by ForumIAS.com – The Knowledge Network for Civil Services.
Visit http//forumias.com New! http://forumiasacademy – Prelims & Mains Test Series

ForumIAS
PRELIMS MARATHON COMPILATION FOR THE MONTH OF MARCH, 2018

b) 2 and 3 only
c) 1, 3 and 4 only
d) 2 and 4 only

Q.15) Which of the following statements is/are correct about Aqua Regia?
1. It is a solution of nitrohydrochloric acid.
2. It is used in hardening of platinum.
Select the correct answer using the code given below.
a) 1 only
b) 2 only
c) Both 1 and 2
d) Neither 1 nor 2

Q.16) Which of the following diseases can be transmitted from one person to another through tattooing?
1. Syphilis
2. Hepatitis B
3. HIV-AIDS
Select the correct answer using the codes given below.
a) 2 and 3 only
b) 3 only
c) 1 and 3 only
d) 1, 2 and 3

Q.17) Consider the following statements:


1. A rainbow is always formed in a direction opposite to that of the Sun.
2. The sunlight undergoes refraction and dispersion in the water droplets to form the colours of a
rainbow.
Which of the above statements is/are correct?
a) 1 only
b) 2 only
c) Both 1 and 2
d) Neither 1 nor 2

Q.18) Which of the following statements is/are correct about Thorium?


1. Thorium is more abundant in nature than uranium.
2. Most common source of thorium is the rare earth phosphate mineral, monazite etc.
3. Thorium is itself fissile and so it can be directly use in a thermal neutron reactor.
Select the correct answer using the code given below.
a) 3 only
b) 1 and 2 only
c) 2 and 3 only
d) 1, 2 and 3

Created with by ForumIAS.com – The Knowledge Network for Civil Services.


Visit http//forumias.com New! http://forumiasacademy – Prelims & Mains Test Series

ForumIAS
PRELIMS MARATHON COMPILATION FOR THE MONTH OF MARCH, 2018

Q.19) Which of the following can be derived from algae?


1. Gasoline
2. Biodiesel
3. Ethanol
4. Jet Fuel
5. Vegetable Oil
6. Butanol
Select the correct answer using the codes given below.
a) 2, 3 and 6 only
b) 1, 2, 3 and 5 only
c) 1, 2 and 4 only
d) All of the above

Q.20) Consider the following statements:


1. Whereas a laser generates a very tight beam, a flashlight produces light that is diffuse.
2. Laser light contains only one wavelength which is determined by the amount of energy released when
the excited electron drops to a lower orbit.
Which of the above statements is/are correct?
a) 1 only
b) 2 only
c) Both 1 and 2
d) Neither 1 nor 2

Q.21) A Hot air balloon will rise to the sky because


1. The balloon is driven by breeze and so there is no control over the horizontal flight path.
2. The Balloon rises to find its equilibrium with the surrounding air.
Which of the above statements is/are correct?
a) 1 only
b) 2 only
c) Both 1 and 2
d) Neither 1 nor 2

Q.22) Which of the following statements is/are correct about Active Pharmaceutical Ingredient (API)?
1. These are secondary ingredient that perform crucial role in delivering a drug to our system.
2. Certain drugs are comprised of more than one kind of API.
Select the correct answer using the code given below.
a) 1 only
b) 2 only
c) Both 1 and 2
d) Neither 1 nor 2

Created with by ForumIAS.com – The Knowledge Network for Civil Services.


Visit http//forumias.com New! http://forumiasacademy – Prelims & Mains Test Series

ForumIAS
PRELIMS MARATHON COMPILATION FOR THE MONTH OF MARCH, 2018

Q.23) Consider the following ingredients:


1. Vinegar
2. Cooking oil
3. Baking soda
Which of the above can be used to neutralize the effect of an ant bite?
a) 3 only
b) 2 and 3 only
c) 1 and 2 only
d) 1, 2 and 3

Q.24) Which of the following statements is/are correct about induction stove?
1. Heating is more uniform than provided by any other method.
2. Cooking occurs due to heat energy produced by the resistance provided by vessel to the eddy currents
generated in the vessel.
3. Copper vessel is not used on such a stove as it is a good conductor; hence no resistance is produced.
Select the correct answer using the code given below.
a) 1 and 3 only
b) 2 only
c) 2 and 3 only
d) 1, 2 and 3

Q.25) Which of the following is/are correct about LED TVs?


1. Their screen is lit by fluorescent tubes.
2. They utilize liquid crystal display technology.
Select the correct answer using the code given below.
a) 1 only
b) 2 only
c) Both 1 and 2
d) Neither 1 nor 2

Q.26) Which of the following statements about the Middle East Respiratory Syndrome (MERS) is
incorrect?
a) It is a viral respiratory disease caused exclusively found in the Middle East region.
b) It was first identified in Saudi Arabia in 2012.
c) It is transmitted through camels and through human-to-human contact.
d) No vaccine or specific treatment for this currently available.

Q.27) What is ‘REPAIR’, recently in the news?


a) Spacecraft repairing vehicle launched by NASA
b) Robotic Servicing of Geosynchronous Satellites (RSGS) program
c) Gene editing tool that can be used to modify DNA in cells
d) Reusable rocket designed by ISRO
Created with by ForumIAS.com – The Knowledge Network for Civil Services.
Visit http//forumias.com New! http://forumiasacademy – Prelims & Mains Test Series

ForumIAS
PRELIMS MARATHON COMPILATION FOR THE MONTH OF MARCH, 2018

Q.28) Which of the following statements is/are correct about electric fuse?
1. The fuse wire has a high melting point.
2. The fuse protects the circuit in case of short circuiting and overloading.
Select the correct answer using the code given below.
a) 1 only
b) 2 only
c) Both 1 and 2
d) Neither 1 nor 2

Q.29) Which of the following are Macronutrients for plants:


1. Magnesium
2. Phosphorous
3. Manganese
4. Calcium
5. Sulfur
6. Molybdenum
Select the correct answer using the codes given below.
a) 1, 3, 4 and 6 only
b) 1, 2, 4 and 5 only
c) 3, 4 and 6 only
d) 1, 2 and 5 only

Q.30) Which of the following statements is/are correct about Neutrinos?


1. It is a subatomic particles with an electric charge.
2. It is produced by the decay of radioactive elements.
Select the correct answer using the code given below.
a) 1 only
b) 2 only
c) Both 1 and 2
d) Neither 1 nor 2

Q.31) Consider the following elements:


1. Water
2. Glucose
3. Carbon Dioxide
Which of the above is/are products of aerobic respiration?
a) 1 and 2 only
b) 2 and 3 only
c) 1 and 3 only
d) 1, 2 and 3

Created with by ForumIAS.com – The Knowledge Network for Civil Services.


Visit http//forumias.com New! http://forumiasacademy – Prelims & Mains Test Series

ForumIAS
PRELIMS MARATHON COMPILATION FOR THE MONTH OF MARCH, 2018

Q.32) Which one of the following describes best the concept of ‘'Die Back' in plant biology?
a) Natural ageing of a plant
b) Seasonal shedding of leaves from a plant
c) Progressive death of twigs and branches which generally starts at the tips
d) Gradual death of a healthy plant due to excessive grazing by animals

Q.33) Which among of the following is/are incorrect about the concept of ‘Botnet’ and ‘Malware’?
1. Botnet is an example of Local Area Network while Malware is a malicious software.
2. A Botnet is a computer security programmme that removes malware.
3. A computer infected with a malware cannot be used in a Botnet.
Select the correct answer using the codes given below.
a) 1, 2 and 3
b) 1 and 2 only
c) 3 only
d) None of the above

Q.34) Which of the following are used as common adulterants in milk?


1. Detergent
2. Hydrogen peroxide
3. Urea
4. Starch
Select the correct answer using the code given below.
a) 1, 2, 3 and 4
b) 1 and 3 only
c) 2 and 3 only
d) 1, 2 and 3

Q.35) Consider the following statements:


1. Transfer of heat by radiation requires a medium.
2. All hot bodies radiate heat.
Which of the statements given above is/are correct?
a) 1 only
b) 2 only
c) Both 1 and 2
d) Neither 1 nor 2

Q.36) Which of the following can be categorized as lifestyle diseases?


1. Type 2 diabetes
2. Heart stroke
3. Colon cancer
4. Atherosclerosis
Select the correct answer the using the code given below:
Created with by ForumIAS.com – The Knowledge Network for Civil Services.
Visit http//forumias.com New! http://forumiasacademy – Prelims & Mains Test Series

ForumIAS
PRELIMS MARATHON COMPILATION FOR THE MONTH OF MARCH, 2018

a) 2, 3 and 4 only
b) 1 and 2 only
c) 2 and 3 only
d) 1, 2, 3 and 4

Answer Key

Qs. Ans. Qs. Ans. Qs. Ans. Qs. Ans.


1 C 11 B 21 C 31 C
2 C 12 A 22 B 32 C
3 A 13 D 23 A 33 A
4 C 14 C 24 B 34 A
5 A 15 A 25 B 35 B
6 B 16 D 26 A 36 D
7 C 17 C 27 C
8 C 18 B 28 B
9 A 19 D 29 B
10 C 20 C 30 B

Explanation
1. Statement 1 is correct. Hydrogen bombs, or thermonuclear bombs, are more powerful than atomic or
"fission" bombs. The difference between thermonuclear bombs and fission bombs begins at the atomic
level.
Statement 2 is correct. Nuclear fusion is a nuclear reaction in which two or more atomic nuclei collide at a
very high energy and fuse together into a new nucleus, e.g. helium.

2. Statement 1 is correct. Graphene is the thinnest compound known to man at one atom thick (a million
times thinner than a human hair), the strongest compound discovered (between 100-300 times stronger
than steel), the lightest material known (with one square meter weighing approximately 0.77
milligrams),and extremely flexible.
Statement 2 is correct. Graphene is impermeable to molecules, and is highly electrically and thermally
conductive – graphene enables electrons to flow much faster than silicon. It is also a transparent
conductor, combining electrical and optical functionalities in an exceptional way.

3. Pair 3 is incorrectly matched.


Phosphoric acid (H3PO4) is used as a rust remover: Phosphoric acid acts as a rust converter, converting
the ferric oxide (Fe2O3) to ferric phosphate (FePO4), which can then be easily scrubbed off. Phosphoric
acid for this purpose is commonly sold as a gel called naval jelly.
Chlorine (Cl2): Chlorine has many uses, including the following:

Created with by ForumIAS.com – The Knowledge Network for Civil Services.


Visit http//forumias.com New! http://forumiasacademy – Prelims & Mains Test Series

ForumIAS
PRELIMS MARATHON COMPILATION FOR THE MONTH OF MARCH, 2018

To produce consumer and industrial products such as plastics, pharmaceuticals, dyes, household cleaners
(including bleach and other disinfecting agents), insecticides, and textiles
To treat water in multiple ways:
To largely eliminate waterborne pathogens during water purification in water treatment plants in the
United States
To kill bacteria in swimming pools; sodium hypochlorite produced from chlorine is used
To act as a major reactant in the production of bulletproof vests, computer chips, and auto parts

4. Statements 1 and 2 are correct.


When light travels from an optically denser medium to a rarer medium at the interface, it is partly
reflected back into the same medium and partly reflected to the second medium. If the angle of incidence
is greater than the critical angle refraction is not possible, and the incident ray is totally reflected. This is
called Total Internal Reflection.

5. Statement 1 is incorrect. A fuel cell produces electricity through a chemical reaction, but without
combustion. It converts hydrogen and oxygen into water, and in the process also creates electricity. It’s an
electro-chemical energy conversion device that produces electricity, water, and heat.
Statement 2 is correct. Hydrogen fuel cells only produce heat and water – no toxins, particles, or
greenhouse gasses, which means clearner air for us to breathe.

6. A photovoltaic cell (PV cell) is a specialized semiconductor diode that converts visible light into direct
current (DC). Some PV cells can also convert infrared (IR) or ultraviolet (UV) radiation into DC electricity.
Photovoltaic cells are an integral part of solar-electric energy systems, which are becoming increasingly
important as alternative sources of utility power.

7. The diopter is the unit of measure for the refractive power of a lens.

8. Statement 1 is correct. Bioluminescence occurs through a chemical reaction that produces light energy
within an organism's body. For a reaction to occur, a species must contain luciferin, a molecule that, when
it reacts with oxygen, produces light.
There are different types of luciferin, which vary depending on the animal hosting the reaction. Many
organisms also produce the catalyst luciferase, which helps to speed up the reaction.
Statement 2 is correct. Bioluminescence is found in many marine organisms: bacteria, algae, jellyfish,
worms, crustaceans, seastars, fish, and sharks to name just a few. In some cases, animals take in bacteria
or other bioluminescent creatures to gain the ability to light up.
For example, the Hawaiian bobtail squid has a special light organ that is colonized by bioluminescent
bacteria within hours of its birth.

9. All of the above events can produce gravitational waves.


Gravitational waves are 'ripples' in the fabric of space-time caused by some of the most violent and
energetic processes in the Universe.
Created with by ForumIAS.com – The Knowledge Network for Civil Services.
Visit http//forumias.com New! http://forumiasacademy – Prelims & Mains Test Series

ForumIAS
PRELIMS MARATHON COMPILATION FOR THE MONTH OF MARCH, 2018

Einstein's mathematics showed that massive accelerating objects (such as neutron stars or black holes
orbiting each other) would disrupt space-time in such a way that 'waves' of distorted space would radiate
from the source (like the movement of waves away from a stone thrown into a pond). Furthermore, these
ripples would travel at the speed of light through the Universe, carrying with them information about
their cataclysmic origins, as well as invaluable clues to the nature of gravity itself.
The strongest gravitational waves are produced by catastrophic events such as colliding black holes, the
collapse of stellar cores (supernovae), coalescing neutron stars or white dwarf stars, the slightly wobbly
rotation of neutron stars that are not perfect spheres, and the remnants of gravitational radiation created
by the birth of the Universe itself.

10. Statement 1 is correct. The Hubble Space Telescope is a large telescope in space. NASA launched
Hubble in 1990.
Statement 2 is correct. The mixture of gases that surround a planet is called its atmosphere. Earth's
atmosphere changes and blocks some of the light that comes from space. Hubble flies around, or orbits,
high above Earth and its atmosphere. So, Hubble can see space better than telescopes on Earth can.

11. Statement 1 is incorrect. Only the cells of advanced organisms, known as eukaryotes, have a nucleus.
Generally there is only one nucleus per cell, but there are exceptions, such as the cells of slime molds and
the Siphonales group of algae. Simpler one-celled organisms (prokaryotes), like the bacteria and
cyanobacteria, don't have a nucleus.
In these organisms, all of the cell's information and administrative functions are dispersed throughout the
cytoplasm.

Statements 2 and 3 are correct. The nucleus is a highly specialized organelle that serves as the
information processing and administrative center of the cell. This organelle has two major functions: it
stores the cell's hereditary material, or DNA, and it coordinates the cell's activities, which include growth,
intermediary metabolism, protein synthesis, and reproduction (cell division).

12. Statement 1 is incorrect. Excitonium is a condensate made up of excitons. Excitons are particles that
are formed in a very strange quantum mechanical pairing. They are obtained by combining escaped
electrons and “holes”.
This quirky quantum-mechanical pairing is possible because, in semiconductors, electrons on edge of one
energy level in atom are able to jump into the next energy level when excited, leaving behind a “hole” in
previous level. This hole acts like a positively charged particle, attracting the negatively charged electron
that escaped.
Statement 2 is correct. Excitonium exhibits macroscopic quantum phenomena, like a superconductor and
this findings holds great promise for unlocking further quantum mechanical mysteries. As a
superconductor and superfluid, this material can be used to further existing technologies. These
applications, especially those in practical technologies, are purely speculative at this point.

13. Exocrine glands are cellular sub-structures, organs, in a body that provide a system to secrete
substances out and external to the body.
Created with by ForumIAS.com – The Knowledge Network for Civil Services.
Visit http//forumias.com New! http://forumiasacademy – Prelims & Mains Test Series

ForumIAS
PRELIMS MARATHON COMPILATION FOR THE MONTH OF MARCH, 2018

They are distinct from the other type of gland, endocrine, in that exocrine gland secretions end up
external to the body, while endocrine secretions go into the bloodstream/internal. Exocrine glands
secrete their substances through a ductal system.
Depending on the exocrine gland, they can function to regulate body temperature, lubricate, nurture
newborns (lactation), aid in digestion, and aid in reproduction.
Examples of Exocrine Glands (and Secreted Product)
Lacrimal gland (Tear ducts and glands near each eye)
Mammary gland (Breast milk)
Eccrine sweat glands (Perspiration or salty water release)
Salivary glands (Saliva consisting of fluid with digestive enzymes)
Pancreas (Pancreatic juice with digestive enzymes into the stomach)
Liver (Bile, green-brown fluid that contains salts and digestion substances)
Note: Examples 5 and 6 are also considered endocrine glands because they also secrete other substances
into the bloodstream

14. Concave mirror is used as a Shaving mirror.


The passenger-side mirror on a car is typically a convex mirror.
These mirrors are often found in the hallways of various buildings (commonly known as "hallway safety
mirrors"), including hospitals, hotels, schools, stores, and apartment buildings.
They are usually mounted on a wall or ceiling where hallways intersect each other, or where they make
sharp turns.
Convex mirrors are used in some automated teller machines as a simple and handy security feature,
allowing the users to see what is happening behind them.
Convex mirrors make everything seem smaller but cover a larger area of surveillance.

15. Statement 1 is correct. Aqua regia, mixture of concentrated nitric and hydrochloric acids. The
traditional solution is comprised of a 3:1 mixture of hydrochloric acid and nitric acid, respectively.
Statement 2 is incorrect.
It is commonly used to remove noble metals such as gold, platinum and palladium from substrates,
particularly in microfabrications and microelectronics labs. Glassware may also be washed with aqua
regia to remove organic compounds only in trace amounts. Aqua regia solutions are extremely corrosive
and may result in explosion or skin burns if not handled with extreme caution.

16. All of the above diseases can be transmitted from one person to another through tattooing.
Syphilis: Syphilis is a sexually transmitted infection (STI) caused by the bacteria Treponema pallidium.
Hepatitis B: Hepatitis B (HBV, Hep B), a virus spread through infected blood or body secretions (blood,
semen, vaginal discharge, breast milk, and saliva).

17. Both the statements are correct.


A rainbow is a natural spectrum appearing in the sky after a rain shower. It is caused by dispersion of
sunlight by tiny water droplets, present in the atmosphere. A rainbow is always formed in a direction
opposite to that of the Sun.
Created with by ForumIAS.com – The Knowledge Network for Civil Services.
Visit http//forumias.com New! http://forumiasacademy – Prelims & Mains Test Series

ForumIAS
PRELIMS MARATHON COMPILATION FOR THE MONTH OF MARCH, 2018

The water droplets act like small prisms. They refract and disperse the incident sunlight, then reflect it
internally, and finally refract it again when it comes out of the raindrop. Due to the dispersion of light
and internal reflection, different colours reach the observer's eye.

18. Statement 1 and 2 are correct. Thorium is a naturally-occurring, slightly radioactive metal discovered
in 1828 by the Swedish chemist Jons Jakob Berzelius, who named it after Thor, the Norse god of thunder.
It is found in small amounts in most rocks and soils, where it is about three times more abundant than
uranium
The most common source of thorium is the rare earth phosphate mineral, monazite, which contains up to
about 12% thorium phosphate, but 6-7% on average. Monazite is found in igneous and other rocks but
the richest concentrations are in placer deposits, concentrated by wave and current action with other
heavy minerals.
Statement 3 is incorrect. Thorium (Th-232) is not itself fissile and so is not directly usable in a thermal
neutron reactor. However, it is ‘fertile’ and upon absorbing a neutron will transmute to uranium-233 (U-
233), which is an excellent fissile fuel materialb. Thus, Thorium fuels need a fissile material as a ‘driver’ so
that a chain reaction (and thus supply of surplus neutrons) can be maintained. The only fissile driver
options are U-233, U-235 or Pu-239.

19. The list of fuels that can be derived from algae includes:
Biodiesel
Butanol
Gasoline
Methane
Ethanol
Vegetable Oil
Jet Fuel
The term third generation biofuel has only recently enter the mainstream it refers to biofuel derived from
algae.

20. Both the statements are correct. Laser light is different from normal. Laser light contains only one
wavelength (one specific color). The particular wavelength of light is determined by the amount of
energy released when the excited electron drops to a lower orbit.
Laser light is directional. Whereas a laser generates a very tight beam, a flashlight produces light that is
diffuse. Because laser light is coherent, it stays focused for vast distances, even to the moon and back.

21. Both the statements are correct. Hot air balloons fly when the air inside the hot air balloon is less
dense than the air surrounding it. Hot air is less dense than cool air; the heated air causes the balloon to
rise simply because it is lighter than an equal volume of cold air. Buoyancy is an upward force that the air
exerts, and it helps hot-air balloons and blimps stay in the air. There will not be any control over the
horizontal flight path.

Created with by ForumIAS.com – The Knowledge Network for Civil Services.


Visit http//forumias.com New! http://forumiasacademy – Prelims & Mains Test Series

ForumIAS
PRELIMS MARATHON COMPILATION FOR THE MONTH OF MARCH, 2018

22. Statement 1 is incorrect. Active Pharmaceutical Ingredients are portions of any drug, which are active.
All drugs are made up of two core components: the API, which is the central ingredient, and the
excipient, the substance inside the drug that helps deliver the medication to your system. Excipients are
chemically inactive substances, such as mineral oil.
For instance, if you have a headache, acetaminophen is the active ingredient, while the liquid in the
capsule or the bulk of a pill is the excipient.
Statement 2 is correct. Certain drugs are comprised of more than one kind of API.

23. The sting of an ant contains formic acid. When an ant bites, it injects the acidic liquid into the skin. The
effect of the sting can be neutralized by rubbing moist baking soda (sodium hydrogen carbonate) or
calamine solution, which contains zinc carbonate as they are basic in nature.

24. Statement 1 is incorrect. Most uniform heating is provided by Microwave Oven.


Statement 2 is correct. In an induction stove cooking occurs due to heat energy produced by the
resistance provided by vessel to the eddy currents generated in the vessel.
Statement 3 is incorrect. Copper vessel is not used because it is not ferromagnetic and hence no eddy
current is produced due to induction. It being a good conductor has no role to play.

25. Statement 1 is incorrect. LCD stands for "liquid crystal display" and technically, both LED and LCD
TVs are liquid crystal displays. The basic technology is the same in that both television types have two
layers of polarized glass through which the liquid crystals both block and pass light. So really, LED TVs
are a subset of LCD TVs.
LED, which stands for "light emitting diodes," differs from general LCD TVs in that LCDs use fluorescent
lights while LEDs use those light emitting diodes. The placement of the lights on an LED TV can differ.
The fluorescent lights in an LCD TV are always behind the screen. On an LED TV, the light emitting
diodes can be placed either behind the screen or around its edges.
The difference in lights and in lighting placement has generally meant that LED TVs can be thinner than
LCDs, although this is starting to change. It has also meant that LED TVs run with greater energy
efficiency and can provide a clearer, better picture than the general LCD TVs. Hence, statement 2 is
correct.

26. Statement (a) is incorrect.


Middle East respiratory syndrome (MERS) is a viral respiratory disease caused by a novel coronavirus
(Middle East respiratory syndrome coronavirus, or MERS‐CoV) that was first identified in Saudi Arabia
in 2012.
Since 2012, 27 countries have reported cases of MERS including Algeria, Austria, Bahrain, China, Egypt,
France, Germany, Greece, Islamic Republic of Iran, Italy, Jordan, Kuwait, Lebanon, Malaysia, the
Netherlands, Oman, Philippines, Qatar, Republic of Korea, Kingdom of Saudi Arabia, Thailand, Tunisia,
Turkey, United Arab Emirates, United Kingdom, United States, and Yemen.
Middle East Respiratory Syndrome (MERS) is spread through human-to-human transmission. However,
current scientific evidence suggests that dromedary camels are a major reservoir host for MERS-CoV and
an animal source of MERS infection in humans.

Created with by ForumIAS.com – The Knowledge Network for Civil Services.


Visit http//forumias.com New! http://forumiasacademy – Prelims & Mains Test Series

ForumIAS
PRELIMS MARATHON COMPILATION FOR THE MONTH OF MARCH, 2018

27. Scientists from The Broad Institute and Massachusetts Institute of Technology in US have developed
RNA Editing for Programmable A to I Replacement (REPAIR), a new gene editing tool therapies that can
reverse disease-causing mutations in humans.
REPAIR gene editing tool can tweak an individual RNA ‘letters’ in human cells without making changes
to entire genome and can have profound potential as a tool for both research and disease treatment.
REPAIR is based on gene editing tool CRISPR (Clustered Regularly Interspaced Short Palindromic
Repeats) that can be used to modify DNA in cells.

28. Statement 1 is incorrect.


Statement 2 is correct.
An electric fuse is a device that is used to protect electric circuits and electric appliances against high
current caused by short - circuiting or overloading due to withdrawal of large current. A fuse is a short
piece of wire made of a material of high resistance and low melting point.

29. Macronutrients constitute Nitrogen (N), Phosphorous (P), Potassium (K), Calcium (Ca), Sulfur (S),
Magnesium (Mg).

30. Statement 1 is incorrect. Neutrinos have zero electric charge.


Statement 2 is correct. Neutrinos are subatomic particles produced by the decay of radioactive elements.

31. Carbon Dioxide and Water are two waste products of aerobic respiration.
Respiration is the process by which people (and other organisms) break down glucose (from sugars and
charbohydrates in their food) in order to release energy. This process requires oxygen which we breath
in.
Aerobic respiration is respiration that uses oxygen as a reactant. Aerobic respiration is much more
efficient, and produces ATP much more quickly, than anaerobic respiration (respiration without oxygen).
This is because oxygen is an excellent electron acceptor for the chemical reaction.

32. "Dieback" refers to the progressive death of twigs and branches which generally starts at the tips.
Trees and shrubs affected by the decline and dieback syndrome may die within a year or two after
symptoms first appear or in some cases survive indefinitely.
Corrective practices such as proper watering, fertilization, and pruning are not guaranteed solutions in all
cases.
Decline and dieback may be caused by many factors and is usually progressive over several years. Trees
and shrubs of all ages may be affected, although this disease complex is usually associated with plants
that have attained some size and maturity.

33. All the statements are incorrect.


A botnet is an interconnected network of computers infected with malware without the user's knowledge
and controlled by cybercriminals. They're typically used to send spam emails, transmit viruses and
engage in other acts of cybercrime. Sometimes known as a zombie army, botnets are often considered one
of the biggest online threats today.
Created with by ForumIAS.com – The Knowledge Network for Civil Services.
Visit http//forumias.com New! http://forumiasacademy – Prelims & Mains Test Series

ForumIAS
PRELIMS MARATHON COMPILATION FOR THE MONTH OF MARCH, 2018

Malware, also known as "malicious software," can be classified several ways in order to distinguish the
unique types of malware from each other. Distinguishing and classifying different types of malware from
each other is important to better understanding how they can infect computers and devices, the threat
level they pose and how to protect against them.
A computer infected with a malware can be used in a Botnet.

34. All are used in adulteration of milk.


“Adulterant” means any material which is or could be employed for making the food unsafe or sub-
standard or misbranded or containing extraneous matter.
Adulterated food is dangerous because it may be toxic and can affect health and it could deprive
nutrients essential for proper growth and development.

35. Radiation is the emission or transmission of energy in the form of waves or particles through space or
through a material medium.
The transfer of heat by radiation does not require any medium. It can take place whether a medium is
present or not. When we come out in the sun, we feel warm. How does the heat from the sun reach us? It
cannot reach us by conduction or convection as there is no medium such as air in most part of the space
between the earth and the sun. From the sun the heat comes to us by another process known as radiation.
Hence, statement 1 is incorrect.
All hot bodies radiate heat. When this heat falls on some object, a part of it is reflected, a part is absorbed
and a part may be transmitted. The temperature of the object increases due to the absorbed part of the
heat. Hence, statement 2 is correct.

36. A disease associated with the way a person or group of people lives. Lifestyle diseases include
atherosclerosis, heart disease, and stroke; obesity and type 2 diabetes; and diseases associated with
smoking and alcohol and drug abuse. Regular physical activity helps prevent obesity, heart disease,
hypertension, diabetes, colon cancer, and premature mortality.
Lifestyle disease: A disease associated with the way a person or group of people lives. Lifestyle diseases
include atherosclerosis, heart disease, and stroke; obesity and type 2 diabetes; and diseases associated
with smoking and alcohol and drug abuse. Regular physical activity helps prevent obesity, heart disease,
hypertension, diabetes, colon cancer, and premature mortality.

Created with by ForumIAS.com – The Knowledge Network for Civil Services.


Visit http//forumias.com New! http://forumiasacademy – Prelims & Mains Test Series

ForumIAS
PRELIMS MARATHON COMPILATION FOR THE MONTH OF MARCH, 2018

Geography And Environment

Q.1) Consider the following statements:


1. The mangrove forests represent an ecotone between marine and terrestrial ecosystem.
2. The density of species like birds is generally lower in the mixed habitat of the ecotone between the
forest and the desert.
Which of the above statements is/are incorrect?
a) 1 only
b) 2 only
c) Both 1 and 2
d) Neither 1 nor 2

Q.2) Consider the following pairs:


1. Bioaccumulation: increase in concentration of a pollutant from the environment to the first organism in
a food chain
2. Biomagnification: increase in concentration of a pollutant from one link in a food chain to another.
Which of the above pairs is/are correctly matched?
a) 1 only
b) 2 only
c) Both 1 and 2
d) Neither 1 nor 2

Q.3) Which of the following animals are generally found in the Thar Desert region of India?
1. Caracal
2. Chinkara
3. Great Indian Bustard
Select the correct answer using the code given below.
a) 1, 2 and 3
b) 3 only
c) 1 and 3 only
d) 2 and 3 only

Q.4) In order for biomagnification to occur, the pollutant must be:


1. Long-lived
2. Mobile
3. Soluble in fats
4. Biologically active
Which of the statements given above is/are correct?
a) 1, 2 and 3 only
b) 2 and 4 only
c) 1 and 3 only

Created with by ForumIAS.com – The Knowledge Network for Civil Services.


Visit http//forumias.com New! http://forumiasacademy – Prelims & Mains Test Series

ForumIAS
PRELIMS MARATHON COMPILATION FOR THE MONTH OF MARCH, 2018

d) 1, 2, 3 and 4

Q.5) The Indian Standard Meridian passes through


1. Madhya Pradesh
2. Karnataka
3. Orissa
4. Uttar Pradesh
Select the correct answer using the codes given below.
a) 1, 3 and 4 only
b) 3 and 4 only
c) 1 and 3 only
d) 1, 2 and 3

Q.6) Consider the following statements:


1. The Aravallis lie parallel to the South West monsoon Arabian Sea branch.
2. The state of Goa receives more rainfall than Aurangabad.
Which of the statements given above is/are correct?
a) 1 only
b) 2 only
c) Both 1 and 2
d) Neither 1 nor 2

Q.7) Consider the following statements:


1. The Brahmaputra River marks the easternmost boundary of the Himalayas.
2. The Eastern Himalayas are wider in extension and covered with dense forests.
Which of the statements given above is/are correct?
a) 1 only
b) 2 only
c) Both 1 and 2
d) Neither 1 nor 2

Q.8) Which of the following statements is/are correct about the TRAFFIC network?
1. It is an intergovernmental organization formed under the United Nations environment Programme.
2. It aims to ensure that trade in wild plants and animals is not a threat to the conservation of nature.
Select the correct answer using the code given below.
a) 1 only
b) 2 only
c) Both 1 and 2
d) Neither 1 nor 2

Q.9) Which of the following statements is/are correct about Asiatic lion?
1. Asiatic lion is listed in Schedule I of Wildlife (Protection) Act 1972.
2. Asiatic lion has been listed as Critically Endangered on IUCN Red List.
Select the correct answer using the code given below.
a) 1 only
Created with by ForumIAS.com – The Knowledge Network for Civil Services.
Visit http//forumias.com New! http://forumiasacademy – Prelims & Mains Test Series

ForumIAS
PRELIMS MARATHON COMPILATION FOR THE MONTH OF MARCH, 2018

b) 2 only
c) Both 1 and 2
d) Neither 1 nor 2

Q.10) The Espoo Convention is primarily associated with


a) Environment Impact Assessment
b) Energy conservation
c) Protection of refugees under international law
d) Emissions trading

Q.11) Consider the following statements:


1. A wildlife sanctuary can be created for a particular species.
2. In a wildlife sanctuary, human activity is completely prohibited.
Which of the above statement is/are correct?
a) 1 only
b) 2 only
c) Both 1 and 2
d) Neither 1 nor 2

Q.12) Which among the following are influenced by photoperiod in an organism?


1. Hibernation
2. Breeding time
3. Migration
Select the correct answer using the code given below.
a) 2 only
b) 1 and 3 only
c) 1 and 2 only
d) 1, 2 and 3

Q.13) Consider the following statements about 'zooxanthellae'?


1. They can be found in jellyfish.
2. They flourish in high temperatures.
Which of the above statements is/are correct?
a) 1 only
b) 2 only
c) Both 1 and 2
d) Neither 1 nor 2

Q.14) Consider the following locations:


1. Gulf of Khambhat
2. Sites of Andaman and Nicobar Islands
3. Gulf of Kutch
4. Creeks of the Sundarban Delta
Which of the above locations are ideal for generation of tidal energy in India?
a) 1 and 4 only
Created with by ForumIAS.com – The Knowledge Network for Civil Services.
Visit http//forumias.com New! http://forumiasacademy – Prelims & Mains Test Series

ForumIAS
PRELIMS MARATHON COMPILATION FOR THE MONTH OF MARCH, 2018

b) 3 and 4 only
c) 2 and 3 only
d) 1, 2, 3 and 4

Q.15) Which of the following are Fold Mountains?


1. Rockies
2. Highlands of Scotland
3. Himalayas
4. Alps
Select the correct answer using the code given below.
a) 1 and 2 only
b) 3 and 4 only
c) 1, 3 and 4 only
d) 1, 2, 3 and 4

Q.16) Which of the following is/are example(s) of Physical Weathering?


1. Block Separation
2. Exfoliation
3. Algae
Select the correct answer using the code given below.
a) 1 only
b) 1 and 2 only
c) 2 and 3 only
d) 1, 2 and 3

Q.17) Consider the following statements about the Mediterranean climate:


1. Shifting of the wind belts is the basic cause of this climate.
2. Mistral and Sirocco are two major hot and dusty winds associated with this climatic.
Which of the above statements is/are correct?
a) 1 only
b) 2 only
c) Both 1 and 2
d) Neither 1 nor 2

Q.18) Which of the following are cold ocean currents?


1. Agulhas current
2. Gulf Stream
3. Californian Current
4. Humboldt Current
5. Benguela Current
Select the correct answer using code given below.
a) 2 and 3 only
b) 2, 3 and 5 only
c) 3, 4 and 5 only
d) 1, 2 and 4 only
Created with by ForumIAS.com – The Knowledge Network for Civil Services.
Visit http//forumias.com New! http://forumiasacademy – Prelims & Mains Test Series

ForumIAS
PRELIMS MARATHON COMPILATION FOR THE MONTH OF MARCH, 2018

Q.19) Which of the following is/are characteristic(s) of savanna landscape?


1. Short and umbrella shaped trees
2. Trees shedding their leaves in the cool, dry season
3. Trees with water-storing devices
Select the correct answer using the codes given below.
a) 1 only
b) 2 and 3 only
c) 1 and 3 only
d) 1, 2 and 3

Q.20) Consider the following climatic conditions:


1. It has high annual range of temperature.
2. It receives rainfall under the influence of South East Monsoon.
3. It is characterized by occurrence of intense tropical cyclones.
Which of the above characteristics are associated with China type of climate?
a) 1 and 3 only
b) 1 and 2 only
c) 2 and 3 only
d) 1, 2 and 3

Q.21) With reference to an initiative called ‘The Biodiversity Finance Initiative (BIOFIN)’, Consider the
following statements:
1. It is managed by The World Wide Fund for Nature (WWF).
2. India's Project Tiger is an outcome of the national obligations towards BIOFIN.
Which of the above statements is/are correct?
a) 1 only
b) 2 only
c) Both 1 and 2
d) Neither 1 nor 2

Q.22) Which of the following statements is/are incorrect about mangrove forest?
1. They do not grow in barrier islands.
2. They require high solar radiation
3. They have the ability to absorb fresh water from saline/brackish water.
4. They produce blind roots.
Select the correct answer using the code given below.
a) 3 only
b) 2 and 4 only
c) 1 only
d) None of the above

Created with by ForumIAS.com – The Knowledge Network for Civil Services.


Visit http//forumias.com New! http://forumiasacademy – Prelims & Mains Test Series

ForumIAS
PRELIMS MARATHON COMPILATION FOR THE MONTH OF MARCH, 2018

Q.23) Consider the following pairs:


Tree Type of Forest
1. Teak: Tropical Evergreen
2. Neem: Tropical Thorn
3. Rosewood: Tropical Deciduous
Which of the pairs given above is/are correctly matched?
a) 2 only
b) 1 and 3 only
c) 2 and 3 only
d) 1, 2 and 3

Q.24) Consider the following states:


1. Madhya Pradesh
2. Gujarat
3. Manipur
4. Jharkhand
5. Tripura
Through which of the above does the Tropic of cancer pass?
a) 2 and 3 only
b) 3 and 5 only
c) 1, 2 4 and 5
d) 1, 2, 3, 4 and 5

Q.25) Which of the following statements is/are correct about the Mediterranean climate?
1. Sunshine is always abundant.
2. Most of the precipitation occurs in the winter.
3. Dense and lofty grasses provides suitable conditions for animal husbandry.
Select the correct answer using the code given below.
a) 3 only
b) 2 and 3 only
c) 1 and 2 only
d) 1, 2 and 3

Q.26) Consider the following pairs:


Mountain Pass State
1. Shipki La: Himachal Pradesh
2. Niti : Sikkim
3. Thang La: Uttarakhand
4. Nathu La: Kashmir
Which of the pairs given above are correctly matched?
a) 1 and 2 only
b) 1 and 3 only
c) 3 and 4 only
d) 2 and 4 only

Created with by ForumIAS.com – The Knowledge Network for Civil Services.


Visit http//forumias.com New! http://forumiasacademy – Prelims & Mains Test Series

ForumIAS
PRELIMS MARATHON COMPILATION FOR THE MONTH OF MARCH, 2018

Q.27) Consider the following pairs:


1. Prairies: North America
2. Pampas: Uruguay
3. Canterbury: Australia
4. Bush-veld: Southern South Africa
Which of the above pairs are correctly matched?
a) 1 and 2
b) 2 and 3
c) 3 and 4
d) 2 and 4

Q.28) Consider the following rivers:


1. Luni
2. Sabarmati
3. Mahi
Which of the above rivers originate(s) from Aravalli Hills?
a) 1 only
b) 2 and 3 only
c) 1 and 2 only
d) 1, 2 and 3

Q.29) Consider the following pairs:


Local term for sacred groves State
1. Deo Bhumi: West Bengal
2. Devara Kadu: Karnataka
3. Jogmaya: Rajasthan
Which of the above pairs is/are correctly matched?
a) 1 only
b) 2 and 3 only
c) 1, 2 and 3
d) None of the above

Q.30) Consider the following pairs:


Population Interaction Feature
1. Mutualism: Both the species are benefited
2. Commensalism: One species is harmed and the other is unaffected
3. Competition: Both the species are harmed
4. Amensalism: One species is benefited and the other is unaffected
Which of the pairs given above is/are correctly matched?
a) 1, 2 and 3 only
b) 1 and 3 only
c) 2 and 4 only
d) 1, 2, 3 and 4

Created with by ForumIAS.com – The Knowledge Network for Civil Services.


Visit http//forumias.com New! http://forumiasacademy – Prelims & Mains Test Series

ForumIAS
PRELIMS MARATHON COMPILATION FOR THE MONTH OF MARCH, 2018

Q.31) Consider the following pollutants:


1. Nitrous Oxide
2. Carbon Dioxide
3. Hydrocarbons
4. Carbon Monoxide
Which of the above pollutants are present in the motor vehicle exhaust emissions?
a) 2, 3 and 4 only
b) 1, 2 and 4 only
c) 1 and 3 only
d) 1, 2, 3 and 4

Q.32) With reference to Glaciers' role in modifying landscape on the Earth’s surface, consider the
following statements:
1. They erode the surface
2. They transport boulders and fine silts
3. They grind rocks, valley bottom and surface
4. They left behind moraines
Which of the above statements is/are correct?
a) 2 and 3 only
b) 1, 2 and 4 only
c) 1, and 2 only
d) 1, 3, 3 and 4

Q.33) Which of the following statements is/are incorrect about the Convention on the Conservation of
Migratory Species of Wild Animals (CMS)?
1. It does not protect marine species and their habitat.
2. It lists migratory species threatened with extinction on Appendix I of the Convention.
Select the correct answer using the codes given below.
a) 1 only
b) 2 only
c) Both 1 and 2
d) Neither 1 nor 2

Q.34) Which of the following statements is/are correct about ecological pyramids?
1. The Pyramid of energy is always upright.
2. The Pyramid of number and biomass can be both upright and inverted.
Select the correct answer using the code given below.
a) 1 only
b) 2 only
c) Both 1 and 2
d) Neither 1 nor 2

Q.35) The 'Aichi Targets' are associated with:


a) Paris Agreement
b) Convention on Biological Diversity
Created with by ForumIAS.com – The Knowledge Network for Civil Services.
Visit http//forumias.com New! http://forumiasacademy – Prelims & Mains Test Series

ForumIAS
PRELIMS MARATHON COMPILATION FOR THE MONTH OF MARCH, 2018

c) Bonn Convention
d) Ramsar Convention

Answer Key

Qs. Ans. Qs. Ans. Qs. Ans. Qs. Ans.


1 B 11 A 21 D 31 D
2 C 12 D 22 C 32 D
3 A 13 A 23 A 33 A
4 D 14 D 24 C 34 C
5 A 15 C 25 C 35 B
6 C 16 B 26 B
7 A 17 A 27 A
8 B 18 C 28 C
9 A 19 D 29 B
10 A 20 D 30 B

Explanation

1. Statement 1 is correct. A zone of junction between two or more diverse ecosystems. For e.g. the
mangrove forests represent an ecotone between marine and terrestrial ecosystem.
Statement 2 is incorrect. Sometimes the number of species and the population density of some of the
species is much greater in this zone than either community. This is called edge effect For example the
density of birds is greater in the mixed habitat of the ecotone between the forest and the desert.

2. Both the statements are correct. Bioaccumulation refers to how pollutants enter a food chain;
biomagnification refers to the tendency of pollutants to concentrate as they move from one trophic level
to the next

3. Thar Desert region of India is home to some of India's most magnificent grasslands and sanctuary for a
charismatic bird, the
Great Indian Bustard. Among the mammal fauna, the blackbuck, wild ass, chinkara, caracal, Sandgrouse
and desert fox inhabit the open plains, grasslands, and saline depressions. The nesting ground of
Flamingoes and the only known population of Asiatic wild Ass lies in the remote part of Great Rann,
Gujarat.

4. If a pollutant is short-lived, it will be broken down before it can become dangerous. If it is not mobile,
it will stay in one place and is unlikely to be taken up by organisms.
If the pollutant is soluble in water it will be excreted by the organism. Pollutants that dissolve in fats,
however, may be retained for a long time.
Created with by ForumIAS.com – The Knowledge Network for Civil Services.
Visit http//forumias.com New! http://forumiasacademy – Prelims & Mains Test Series

ForumIAS
PRELIMS MARATHON COMPILATION FOR THE MONTH OF MARCH, 2018

It is traditional to measure the amount of pollutants in fatty tissues of organisms such as fish. In
mammals, we often test the milk produced by females, since the milk has a lot of fat in it and because the
very young are often more susceptible to damage from toxins (poisons).
If a pollutant is not active biologically, it may biomagnify, but we really don't worry about it much, since
it probably won't cause any problems.
5. Indian Standard Meridian (82˚30’E meridian) passes through UP, MP, Chhattisgarh, Orissa and
Andhra Pradesh.

6. Statement 1 is correct. In spite of the Aravalli hills, many parts of Rajasthan, Western Rajasthan
/Bikaner/Jaisalmer / Jodhpur /Pokaran do not receive much rainfall. This is because the Aravallis lie
parallel to the South West monsoon Arabian Sea branch.
For this reason the winds by pass this region and no rain is shed as there are no mountain ranges that lie
transversely across. More over this region is also on the leeward side or in the Rain shadow area /region
when the South West Monsoon Bay of Bengal branch blows.
Statement 2 is correct. Mumbai, Goa, Panaji, Mangalore, Cochin, Thiruvananthapuram receive more
rainfall than Pune, Sholapur, Nagpur, Amravathi, Aurangabad. Because Mumbai, Goa, Panaji,
Mangalore, Cochin, Thiruvananthapuram lie on the Windward side of the Western Ghats while Pune,
Sholapur, Nagpur, Amravathi, Aurangabad lie on the Leeward side or in the Rain Shadow region and
hence when the South West Monsoon Arabian Sea Branch blows, they receive heavy rain.

7. Statement 1 is correct. Eastern Himalayas: This part of the Himalayas lies between the Tista River in
the west and the Brahmaputra River in the east and stretches for a distance of about 720 km.
Statement 2 is incorrect. The Eastern Himalayas are narrower than its northern counterpart. However,
the Eastern Himalayas are covered with 'dense forests.
The northern slopes of the hills in Himalayan region are at high altitude and have low temperature or are
snow covered. This does not allow much vegetation. In the southern part of the hills there are plains and
valleys watered by rivers and with fertile soil and pleasant temperature levels all required for dense
growth of vegetation.

8. Statement 1 is incorrect. TRAFFIC is a joint conservation programme of WWF and IUCN. TRAFFIC,
the wildlife trade monitoring network, is the leading non-governmental organization working globally
on trade in wild animals and plants in the context of both biodiversity conservation and sustainable
development.
The United Nations Environment Programme is an agency of United Nations and coordinates its
environmental activities, assisting developing countries in implementing environmentally sound policies
and practices.
Statement 2 is correct. TRAFFIC’s mission is to ensure that trade in wild plants and animals is not a
threat to the conservation of nature.
TRAFFIC was established in 1976 in what remains a unique role as a global specialist, leading and
supporting efforts to identify conservation challenges and support solutions linked to trade in wild
animals and plants.
TRAFFIC is governed by the board of trustees of TRAFFIC International. We work particularly closely
with our founding organisations, IUCN and WWF, making a critical contribution to achievement of their
conservation goals through a unique partnership that complements and engages the considerable
strengths of each of these two major global conservation organizations.
Created with by ForumIAS.com – The Knowledge Network for Civil Services.
Visit http//forumias.com New! http://forumiasacademy – Prelims & Mains Test Series

ForumIAS
PRELIMS MARATHON COMPILATION FOR THE MONTH OF MARCH, 2018

9. Statement 1 is correct.
Statement 2 is incorrect. Listed in Schedule I of Wildlife (Protection) Act 1972, in Appendix I of CITES
and as Endangered on IUCN Red List.

10. The Espoo Convention sets out the obligations of Parties to assess the environmental impact of certain
activities at an early stage of planning. It also lays down the general obligation of States to notify and
consult each other on all major projects under consideration that are likely to have a significant adverse
environmental impact across boundaries.

11. Wildlife sanctuary can be created for a particular specie,s (for e.g. grizzled giant squirrel) whereas the
national park is not primarily focused on a particular species. Hence, statement 1 is correct.
Any area other than area comprised with any reserve forest or the territorial waters can be notified by the
State Government to constitute as a sanctuary if such area is of adequate ecological, faunal, floral,
geomorphological, natural. or zoological significance, for the purpose of protecting, propagating or
developing wildlife or its environment.
Statement 2 is incorrect. Human activities like tilling of land, collection of minor forest produce,
harvesting of timber and private ownership rights are allowed in wildlife sanctuary.

12. Photoperiod refers to the time that a plant or animal is exposed to light in a 24-hour period.
Hibernation, Breeding time and Migration of living organisms in different habitat is influenced by the
Photoperiod.

13. Statement 1 is correct.


Zooxanthellae are tiny plant-like organisms called zooxanthellae live in the tissues of many animals,
including some corals, anemones, and jellyfish, sponges, flatworms, mollusks and foraminifera.
These microscopic algae capture sunlight and convert it into energy, just like plants, to provide essential
nutrients to the corals. In exchange, they have a place to live inside the animal's body. But when the
zooxanthellae are under stress, such as high temperatures, they will die or leave their host—a process
known as bleaching. Hence, statement 2 is incorrect.

14. All of the above locations are ideal for generation of tidal energy.

15. Fold Mountains occur near convergent or compressional plate boundaries. Examples of Fold
Mountains include the Alps, Rockies, Andes and Himalayas.
The highland of Scotland are Scandinavia are residual mountains.

16. Block Separation and Exfoliation are examples of Physical Weathering.


Action of Algae is an example of Biological weathering.
1. Exfoliation: With rise in temperature, every mineral expands and pushes against its neighbor and as
temperature falls, a corresponding contraction takes place. Because of diurnal changes in the
temperatures, this internal movement among the mineral grains takes place regularly. This process is
most effective in dry climates and high elevations where diurnal temperature changes are drastic.

Created with by ForumIAS.com – The Knowledge Network for Civil Services.


Visit http//forumias.com New! http://forumiasacademy – Prelims & Mains Test Series

ForumIAS
PRELIMS MARATHON COMPILATION FOR THE MONTH OF MARCH, 2018

2. Block Separation: This type of disintegration takes place in rocks with numerous joints acquired by
mountain-making pressures or by shrinkage due to cooling. This type of disintegration in rocks can be
achieved by comparatively weaker forces.

17. Statement 1 is correct. The Mediterranean climate entirely confined to the western portion of
continental masses, between 30° and 45° north and south of the equator. The basic cause of this type of
climate is the shifting of the wind belts. In summer when the sun is overhead at the Tropic of Cancer, the
belt of influence of the Westerlies is shifted a little pole wards. Rain bearing winds are therefore not likely
to reach the Mediterranean lands. The prevailing Trade Winds [tropical easterlies] are off-shore and there
is practically no rain. The Mediterranean lands receive most of their precipitation in winter when the
Westerlies shift equator wards.
In the northern hemisphere, the prevailing on-shore Westerlies bring much cyclonic rain from the
Atlantic.
Thus, the Mediterranean climate has the characteristics of clear skies and high temperatures; hot, dry
summers and cool, wet winters.
Statement 2 is incorrect. Although both the winds i.e. Sirocco and Mistral are associated with
Mediterranean climate, they are totally different in nature. While Sirocco is a hot, dry dusty wind, Mistral
is a cold wind.

18. Agulhas is a warm oceanic current of Southwest Indian Ocean.


Gulf Stream is a warm current which flows south-east of USA.
California current is cold current which flows along the coasts of the western USA.
Humboldt Current or Peruvian current is cold current which flows along the west coast of South
America.
Benguela Current in Atlantic Ocean is a cold current.

19. All of the above are characteristics of savanna landscape.


The savanna landscape is characterized with tall and luxuriant grass. In true savanna lands, the grass is
coarse, growing 6 to 12 feet high. The elephant grass may attain a height of even 15 feet. The trees in
savanna landscape are short in height. Many trees are umbrella shaped, exposing only a narrow edge to
the strong winds.
In Savanna landscape, the trees are deciduous, shedding their leaves in the cool, dry season to prevent
excessive loss of water through transpiration, e.g. Acacias.
Some trees found in the Savanna landscape have broad trunks, with water-storing devices to survive
through the prolonged drought such as baobabs and bottle trees. These are mostly hard, gnarled and
thorny and may exude gum like gum Arabic.

20. All of the above are characteristics of China type of climate.


China type climate experiences South East monsoon during summer with heavy rainfall and North West
monsoon during winter with dry and cold weather.
It has high annual temperature range. During winter a steep pressure gradient is set up between the cold
Mongolia and Siberia and the warmer Pacific coastlands. This leads to the flow of continental polar
streams.
It is characterized by the formation of tropical cyclone in southern China. E.g. typhoons. The cyclones
originate in the Pacific Ocean and move westward to the coastal lands bordering the South China Sea.
Created with by ForumIAS.com – The Knowledge Network for Civil Services.
Visit http//forumias.com New! http://forumiasacademy – Prelims & Mains Test Series

ForumIAS
PRELIMS MARATHON COMPILATION FOR THE MONTH OF MARCH, 2018

21. Statement 1 is incorrect. BIOFIN is managed by UNDP, in partnership with the European
Commission and the Governments of Germany, Switzerland, Norway and Flanders.
Statement 2 is incorrect. Project Tiger is a tiger conservation programme launched in 1973 by the
Government of India. This project does not have direct association with BIOFIN. The project tiger aims
at ensuring a viable population of Bengal tigers in their natural habitats and also to protect them from
extinction, and preserving areas of biological importance as a natural heritage forever represented as
close as possible the diversity of ecosystems across the tiger's distribution in the country.

22. Statement 1 is incorrect. They are basically evergreen land plants growing on sheltered shores,
typically on tidal flats, deltas, estuaries, bays, creeks and the barrier islands. The best locations are where
abundant silt is brought down by rivers or on the backshore of accreting sandy beaches.
Statements 2, 3 and 4 are correct. Mangroves require high solar radiation and have the ability to absorb
fresh water from saline/ brackish water. Mangrove produces pneumatophores (blind roots) to overcome
respiratory problem in the anaerobic soil conditions.

23. Pair 2 is correctly matched.


Tropical Thorn Forests: Important species found are babool, ber, and wild date palm, khair, neem, khejri,
palas, etc. Tropical thorn forests occur in the areas which receive rainfall less than 50 cm.
Tropical Evergreen and Semi Evergreen Forests: Species found in these forests include rosewood,
mahogony, aini, ebony, etc. These forests are found in the western slope of the Western Ghats, hills of the
north-eastern region and the Andaman and Nicobar Islands.
Tropical Deciduous Forests: These are the most widespread forests in India. They are called the Monsoon
forests. Teak, sal, shisham, hurra, mahua, amla, semul, kusum, and and salwood etc. are the main species
of these forests.

24. Tropic of cancer passing through Indian states are as follows


1-Gujarat
2- Rajasthan
3- Madhya Pradesh
4- Chattisgarh
5- Jharkhand
6- West Bengal
7- Tripura
8- Mizoram

25. Statement 1 is correct. Bright, sunny weather with hot dry summers and wet, mild winters are main
characteristics of the Mediterranean region.
In the Mediterranean region, the sky is almost cloudless and sunshine is always abundant. Even, in mid-
summer, the intense heat is never sultry.
Statement 2 is correct. The Mediterranean lands receive most of their precipitation in winter when the
westerlies shift equatorwards.

26. The correct arrangement is as follows:


1. Shipki La pass: Himachal Pradesh
Created with by ForumIAS.com – The Knowledge Network for Civil Services.
Visit http//forumias.com New! http://forumiasacademy – Prelims & Mains Test Series

ForumIAS
PRELIMS MARATHON COMPILATION FOR THE MONTH OF MARCH, 2018

2. Niti Pass: Uttarakhand


3. Thang La: Uttarakhand
4. Nathu La: Sikkim

27. Pairs 1 and 2 are correctly matched.


Canterbury is found in New Zealand and Bush-veld is found in Northern South Africa.

28. Luni and Sabarmati rivers originate from the Aravallis. Mahi originates from
Vindhyachal Hills.

29. Pairs 2 and 3 are correctly matched.


Pair 1 is incorrectly matched. Deo Bhumi is a term used to refer sacred groves in Uttarakhand.
The sacred groves in West Bengal are called Garamthan, Harithan, Jahera, Sabitrithan, Santalburithan etc.

30. Pair 1 is correctly matched. Mutualism is the interaction in which both the interacting species benefit,
for example, the association between fungi and the roots of higher plants. The fungi help the plant in the
absorption of essential nutrients from the soil while the plant in turn provides the fungi with energy-
yielding carbohydrates.
Pair 2 is incorrectly matched. Commensalism is the interaction among species in which one species
benefits and the other is neither harmed nor benefited, for example, an orchid growing as an epiphyte on
a mango branch. In this interaction while orchid benefits, the mango tree remains unaffected.

Pair 3 is correctly matched. Competition is the interaction in which species compete with each other to
utilize the limited available resource. In this interaction both the species lose as they try to outdo each
other, for example, the herbivores animals compete with each other to access limited grazing resources.
Pair 4 is incorrectly matched. Amensalism is the interaction in which one species is harmed while the
other is unaffected, for example, some higher plants like black walnut secrete substances that inhibit the
growth of or kill the nearby competing plants.

31. All of the above pollutants are present in the motor vehicle exhaust emissions.
They are part of exhaust emissions produced by motor vehicles. Passenger vehicles are a major pollution
contributor, producing significant amounts of nitrogen oxides, carbon monoxide, and other pollution.
An exhaust gas is a gas emitted through a combustion process. The exhaust gas is actually a combination
of many different gases: N2, CO2, H2O and O2. Though some are harmless, there are few that are
harmful and are considered as major pollutants. For example: CO (carbon monoxide).

32. All the statements are correct. Glaciers modify the landscape in a number of ways. They erode the
surface, transport and deposit materials of all sizes –ranging from boulders and sand to fine silt.
A glacier armed with pieces of rocks grinds the valley bottom as well as its sides. As a result a valley
occupied by a glacier has flat bottoms and steep or vertical sides. As a glacier melts, it leaves mounds of
rock debris at its edge. Such mounds of unsorted rock materials left behind by a glacier is called a
moraine.

33. Statement 1 is incorrect. CMS provides a global platform for the conservation and sustainable use of
migratory animals (terrestrial, marine and avian migratory species) and their habitats. CMS brings
Created with by ForumIAS.com – The Knowledge Network for Civil Services.
Visit http//forumias.com New! http://forumiasacademy – Prelims & Mains Test Series

ForumIAS
PRELIMS MARATHON COMPILATION FOR THE MONTH OF MARCH, 2018

together the States through which migratory animals pass, the Range States, and lays the legal foundation
for internationally coordinated conservation measures throughout a migratory range.
Statement 2 is correct. Migratory species threatened with extinction are listed on Appendix I of the
Convention. CMS Parties strive towards strictly protecting these animals, conserving or restoring the
places where they live, mitigating obstacles to migration and controlling other factors that might
endanger them. Besides establishing obligations for each State joining the Convention, CMS promotes
concerted action among the Range States of many of these species.

34. Both the statements are correct.


Pyramid of energy is always upright, can never be inverted, because when energy flows from a particular
trophic level to the next trophic level, some energy is always lost as heat at each step.
Each bar in the energy pyramid indicates the amount of energy present at each trophic level in a given
time or annually per unit area.
Ecological Pyramid of number and biomass can be both upright and inverted. For example Pyramid of
numbers in forest and parasitic food chain is upright and inverted respectively.
Pyramid of biomass is upright in forest ecosystem because the biomass of producer is much more than
herbivores or consumer which outweigh the biomass of upcoming trophic level (carnivores). However in
pond ecosystem, pyramid of biomass is inverted since the producer biomass is less than primary and
secondary consumer (biomass of large fish is more than phytoplankton or small producers).

35. The 'Aichi Targets' are associated with Convention on Biological Diversity.
The Aichi Targets are a set of 20, time-bound, measureable targets for the conservation of biodiversity.
The Strategic Plan is comprised of a shared vision, a mission, strategic goals and 20 ambitious yet
achievable targets, collectively known as the Aichi Targets.

Created with by ForumIAS.com – The Knowledge Network for Civil Services.


Visit http//forumias.com New! http://forumiasacademy – Prelims & Mains Test Series

ForumIAS

You might also like